EAQs Fundamentals: Health and Physical Assessment

Ace your homework & exams now with Quizwiz!

The nurse is assessing a client following abdominal surgery. Which assessment findings should the nurse use to form a data cluster? Select all that apply.

- The client reports pain with movement. - The client has pain over the surgical area. - The client rates the pain as 8 on a scale of 0 to 10.

The depth of indentation left after applying pressure to an edematous site determines the degree of edema. A 1+ score is given if the depth of indentation is: A 2+ is the score given if the depth of edema indentation is:

2 mm. 4 mm.

The nurse cares for an unconscious client who underwent head surgery. Which site would be best used to monitor body temperature? 1 Skin 2 Oral 3 Axilla 4 Rectal

4 Rectal Although the oral route is the most common route for monitoring body temperature, clients who are unconscious should have their temperatures monitored rectally. Skin temperature may be impaired due to diaphoresis; this measurement may not reliable. The axilla temperature may underestimate the core temperature.

An infant usually doubles his or her birth weight at

4 to 5 months of age.

While assessing a client, a nurse finds that the ratio of the anteroposterior diameter and transverse diameter of the chest is 1:1. What is indicated by this finding? Select all that apply.

Client is an older adult Client has a history of smoking. Client has chronic lung disease

SKIN COLOR VARIATIONS: Bluish (cyanosis)

Increased amt of deoxygenated hemoglobin (associated w/ hypoxia) Causes: Heart or lung disease, cold environment. Assessment locations: Nail beds, mouth, skin, lips

What are the desired outcomes that the nurse expects when administering a nonsteroidal antiinflammatory drug (NSAID)?

Pain relief, antipyresis, reduced inflammation

A congenital condition in which both eyes do not focus on an object simultaneously, these eyes appear crossed. Impairment of the extraocular muscles or their nerve supply cause this

Strabismus

nerve helps in the eyeball's lateral movement.

The abducens *CNVI*(6)

provides sensory and motor innervations for facial expressions.

The facial nerve *CNVII*(7)

represents the systolic pressure in children and adults.

The first Korotkoff sound

Ulnar side of forearm at wrist. Site used to assess status of circulation to hand; also used to preform an Allen's test

Ulnar Pulse

a high-pitched sound that may be caused by a high velocity airflow through an obstructed or narrowed airway.

Wheezing is

a condition in which the opacity of the lens will be increased; this disorder is commonly related to age.

A cataract is

What should the nurse consider when obtaining an informed consent from a 17-year-old adolescent?

If the client is allowed to give consent

Which steps are taken by a nurse during the implementation phase of drug research?

Obtaining necessary approvals & Recruiting subjects and collecting data obtained from the study

Muscular spasms in larger airways or any new growth causing turbulence may produce

rhonchi, which is a loud and low-pitched sound.

No evidence of muscle contractility

Grade: 0 %: 0 Lovett scale: 0 (zero)

A nurse is teaching staff members about the legal terminology used in child abuse. What definition of battery should the nurse include in the teaching?

The application of force to another person without lawful justification.

The 1:1 ratio of the anteroposterior diameter and transverse diameter of the chest indicates

a barrel-shaped chest.

Inflammation of the pleura may produce

pleural friction rubs.

A client is scheduled to receive phenytoin (Dilantin) 100 mg orally at 6 PM but is having difficulty swallowing capsules. What method should the nurse use to help the client take the medication?

Administer 4 mL of phenytoin suspension containing 125 mg/5 mL

The nurse recognizes that which are important components of a neurovascular assessment are:

Orientation, Respiratory rate, Pulse and skin temperature

The nurse is caring for a client that is hyperventilating. The nurse recalls that the client is at risk for:

Respiratory alkalosis

A client is admitted with metabolic acidosis. The nurse considers that two body systems interact with the bicarbonate buffer system to preserve healthy body fluid pH. What two body systems should the nurse assess for compensatory changes?

Respiratory and urinary

What are the best ways for a nurse to be protected legally?

Provide care within the parameters of the state's nurse practice act, document consistently and objectively, Clearly document a client's non-adherence to the medical regimen

A registered nurse is teaching a nursing student how to assess for edema. Which statement made by the student indicates the need for further education?

"If the pressure on an edematous site leaves an indentation of 2 mm, a score of 2+ is given.

Full range of motion against gravity, some resistance

Grade: 4 %: 75 Lovett: g (good)

The inner side of the ankle, below medial malleolus. the site used to assess status to circulation to foot

Posterior tibal

Easily accessible site used to assess pulse in children. Over temporal bone in head, above and lateral to eye

Temporal pulse

there is an increase in lumbar curvature.

lordosis

The client with a change in body image following an injury recognizes the reality of the change, becomes anxious, and refuses to discuss it. This client uses ________as an adaptive coping mechanism.

withdrawl

The nurse is caring for a client whose forehead feels warm to the touch. The nurse uses a thermometer and obtains the client's temperature. What is the nurse doing? 1 Validation 2 Assessment 3 Interpretation 4 Documentation

1. Validation The nurse is validating the presence of fever in the client. Validation is the process of gathering more assessment data. It involves clarifying vague or unclear data. Assessment is the first step of the nursing process. It involves collecting information from the client and secondary sources. During interpretation, the nurse recognizes that further observations are needed to clarify information. Data documentation is the last part of a complete assessment. The nurse must document facts in a timely, thorough, and accurate manner to prevent information from getting lost.

The nurse is caring for a client with a closed soft tissue injury. The nurse describes the injury as a/an:

Contusion

A nurse is caring for a client on bed rest. How can the nurse help prevent a pulmonary embolus?

Teach the client how to exercise the legs.

A nurse is preparing to administer an ophthalmic medication to a client. What techniques should the nurse use for this procedure?

Clean the eyelid and eyelashes, apply clean gloves before beginning of procedure, Press on the nasolacrimal duct after instilling the solution

A nurse is evaluating the effectiveness of treatment for a client with excessive fluid volume. What clinical finding indicates that treatment has been successful?

Clear breath sounds

The nurse is discussing discharge plans with a client who had a myocardial infarction. The client states, "I'm worried about going home." The nurse responds, "Tell me more about this." What interviewing technique did the nurse use?

Exploring

A client with arthritis increases the dose of ibuprofen (Motrin, Advil) to abate joint discomfort. After several weeks the client becomes increasingly weak. The client is admitted to the hospital and is diagnosed with severe anemia. What clinical indicators does the nurse expect to identify when performing an admission assessment?

Melena, Tachycardia

What is the maximum length of time a nurse should allow an intravenous (IV) bag of solution to infuse?

24 Hours

While performing a physical assessment of a female client, a nurse notices hair on the client's upper lip, chin, and cheeks. Which condition may result in this condition? 1 Aging 2 Poor nutrition 3 Endocrine disease 4 Arterial insufficiency

3 Endocrine disease Endocrine diseases such as hirsutism will result in excessive hair growth on the upper lip, chin, and cheeks. Aging and poor nutrition will result in decreased hair growth. Arterial insufficiency will result in decreased hair growth due to compromised blood supply.

A nurse administers an intravenous solution of 0.45% sodium chloride. In what category of fluids does this solution belong? 1 Isotonic 2 Isomeric 3 Hypotonic 4 Hypertonic

3 Hypotonic Hypotonic solutions are less concentrated (contain less than 0.85 g of sodium chloride in each 100 mL) than body fluids. Isotonic solutions are those that cause no change in the cellular volume or pressure, because their concentration is equivalent to that of body fluid. This relates to two compounds that possess the same molecular formula but that differ in their properties or in the position of atoms in the molecules (isomers). Hypertonic solutions contain more than 0.85 g of solute in each 100 mL.

While auscultating the heart, a healthcare provider notices S3 heart sounds in four clients. Which client is at more risk for heart failure? 1 Child client 2 Pregnant client 3 Older adult client 4 Young adult client

3 Older adult client The S3 is the third heart sound heard after the normal "lub-dub." It is indicative of congestive heart failure in adults over 30 years old. In young, pregnant, and under 30 year old clients, the third heart sound is often considered to be a normal parameter. Test-Taking Tip: You have at least a 25% chance of selecting the correct response in multiple-choice items. If you are uncertain about a question, eliminate the choices that you believe are wrong and then call on your knowledge, skills, and abilities to choose from the remaining responses.

While performing a physical assessment in a client, the registered nurse (RN) notices reddish linear streaks in the nail bed. Which systemic condition can the registered nurse (RN) suspect in the client based on these assessment findings?

3. Subacute bacterial endocarditis Red or brown linear streaks in the nail bed are caused by minor trauma to nails, subacute bacterial endocarditis, or trichinosis and are called splinter hemorrhages.

A registered nurse is teaching a nursing student how to assess for edema. Which statement made by the student indicates the need for further education? 1 "Edema results in the separation of skin from pigmented and vascular tissue." 2 "Pitting edema leaves an indentation on the site of application of pressure." 3 "Trauma or impaired venous return should be suspected in clients with edema." 4 "If the pressure on an edematous site leaves an indentation of 2 mm, a score of 2+ is given."

4 "If the pressure on an edematous site leaves an indentation of 2 mm, a score of 2+ is given." The depth of indentation left after applying pressure to an edematous site determines the degree of edema. A 1+ score is given if the depth of indentation is 2 mm. A 2+ is the score given if the depth of edema indentation is 4 mm. An accumulation of edematous fluid will result in the separation of skin and underlying vasculature. Edema is classified as pitting if the application of pressure on the edematous site will leave an indentation for some time. Edema results from a direct trauma to the tissue or by impaired venous return.

A nurse is assigned to care for a college student who has been talking to unseen people and refusing to get out of bed, go to class, or participate in daily grooming activities. What is the nurse's initial effort toward helping this client? 1 Providing frequent rest periods 2 Reducing environmental stimuli 3 Facilitating the client's social relationships with a peer group 4 Attempting to establish a meaningful relationship with the client

4 Attempting to establish a meaningful relationship with the client The first step in a plan of care should be the establishment of a meaningful relationship because it is through this relationship that the client can be helped. This client is not getting out of bed; rest periods are not needed. The client has already reduced environmental stimuli by staying in bed; further reduction is not needed. Establishing social relationships is a long-term goal.

The nurse is performing a weight assessment for different people in a community. Which question should the nurse ask a client to determine a disease-related change in weight? 1 Do you follow a strict calorie intake? 2 Have you notices any changes in the social aspects of eating? 3 Are you taking diuretics or insulin? 4 Have you noticed any unintentional weight loss in the past six months?

4 Have you noticed any unintentional weight loss in the past six months? Unintentional or undesired weight loss during a certain period of time may indicate a weight change due to a disease, such as gastrointestinal problems. A strict calorie intake in a permitted limit is not related to any disease. Assessing the social aspects of a client's eating habits determines any lifestyle changes that may cause a weight change. Diuretics and insulin may cause weight loss or weight gain; this change is not disease-related.

Full Fowlers position

90 degree angle

A nurse assesses a client with dry and brittle hair, flaky skin, a beefy-red tongue and bleeding gums. The nurse recognizes that these clinical manifestations are most likely a result of:

A nutritional deficiency.

Trendelenburg Position (shock position)

A position in which client is supine with the legs below the level of the heart. .

The family of an older adult who is aphasic reports to the nurse manager that the primary nurse failed to obtain a signed consent before inserting an indwelling catheter to measure hourly output. What should the nurse manager consider before responding?

A separate signed informed consent for routine treatments is unnecessary

A nurse is caring for a client admitted with cardiovascular disease. During the assessment of the client's lower extremities, the nurse notes that the client has thin, shiny skin, decreased hair growth, and thickened toenails. The nurse understands that this may indicate:

Arterial Insufficiency

the first step of the nursing process. It involves collecting information from the client and secondary sources.

Assessment is

A nurse is caring for an elderly client with dementia who has developed dehydration as a result of vomiting and diarrhea. Which assessment best reflects the fluid balance of this client?

Blood lab results

SKIN COLOR VARIATIONS: Red Erythema

Condition: Increases visibility of oxyhemoglobin caused by dilation or increased blood flow Causes: Fever, direct trauma, blushing, alcohol intake Assessment location: Face, the area of trauma, etc

SKIN COLOR VARIATIONS: Loss of pigmentation

Condition: Vitiligo Causes: Congenital or autoimmune condition causing lack of pigment Assessment location: Patchy area of skin over the body

What is a nurse's responsibility when administering prescribed opioid analgesics?

Count the client's respirations, document the intensity of the client's pain, verify the number of doses in the locked cabinet before administering the prescribed dose

When performing a postoperative assessment, which parameter would alert the nurse to a common side effect of epidural anesthesia?

Decreased blood pressure

A nurse reviews a medical record of a client with ascites. What does the nurse identify that may be causing the ascites?

Decreased liver function

Turgor is the elasticity of the skin. Poor skin turgor could indicate:

Dehydration

Along top pf foot, b/t tendons of great and first toe. Site used to assess circ to foot

Dorsalis pedis pulse

A health care provider has prescribed isoniazid (Laniazid) for a client. Which instruction should the nurse give the client about this medication?

Drinking alcohol daily can cause drug-induced hepatitis

a common eye disorder among people over 65. It causes 1. blurred or reduced central vision, due to thinning of the macula 2. symptoms usually develop gradually and without pain. 3. Visual distortions, such as straight lines seeming bent 4. Reduced central vision in one or both eyes 5. The need for brighter light when reading or doing close work 6. Increased difficulty adapting to low light levels, such as when entering a dimly lit restaurant 7. Increased blurriness of printed words 8. Decreased intensity or brightness of colors 9. Difficulty recognizing faces

Dry macular degeneration

The nurse is caring for a client who requires an intravenous infusion. The nurse explains the reason for the procedure while assembling the kit for the infusion. What is the role of the nurse in this situation?

Educator

A nurse is explaining the nursing process to a nursing assistant. Which step of the nursing process should include interpretation of data collected about the client?

Evaluation

Below the inguinal ligament, midway b/t symphysis pubis and anterior superior iliac spine. The site used to assess character of a pulse during physiological shock or cardiac arrest when other pulses are not palpable; used to assess status of circulation to leg

FEMORAL PULSe

A nurse is caring for a postoperative client who had general anesthesia during surgery. What independent nursing intervention may prevent an accumulation of secretions?

Frequent changes of position

The nurse is teaching a client about adequate hand hygiene. What component of hand washing should the nurse include that is most important for removing microorganisms?

Friction

Slight muscle contractility, no movement

Grade: 1 %: 10 Lovett: T (trace)

Full range of motion, gravity eliminated

Grade: 2 %: 25 Lovett scale: P (poor)

Full range of motion with gravity

Grade: 3 %: 50 Lovett: f (fair)

Full range of motion against gravity full resistance

Grade: 5 %: 100 Lovett: n (normal)

What rules of impression management should the nurse follow when caring for an Asian client?

Greet the client and family in their language. Observe the distance maintained by the client. Clarify whether the client wants someone from the family to be present.

A nurse is providing preoperative teaching for a client regarding use of an incentive spirometer and should include what instructions?

Inhale deeply through the spirometer, hold it as long as possible, and slowly exhale

Blurred central vision, often occurring suddenly caused by progressive degeneration of the retina. Most common visual impairment of Indy over the age of 50 and the most common cause of blindness in adults. There is no cure

Macular degeneration

A health care provider prescribes transdermal fentanyl (Duragesic) 25 mcg/hr every 72 hours. During the first 24 hours after starting the fentanyl, what is the most important nursing intervention?

Manage pain with oral pain medication

A nurse is obtaining a health history from the newly admitted client who has chronic pain in the knee. What should the nurse include in the pain assessment?

Pain history including location, intensity and quality of pain;Pain pattern including precipitating and alleviating factors

Alternative therapy measures have become increasingly accepted within the past decade, especially in the relief of pain. Which methods qualify as alternative therapies for pain?

Prayer, hypnosis, aromatherapy, guided imagery

A nurse is collecting a client's medical information by talking to the family and caregivers of the client. Which phase of helping relationships is the nurse exercising?

Preinteraction phase

Impaired near vision in middle-age and older adults, caused by *loss of elasticity of the lens* and is associated with the aging process

Presbyopia

A nurse must establish and maintain an airway in a client who has experienced a near-drowning in the ocean. For which potential danger should the nurse assess the client?

Pulmonary edema

The nurse cares for an unconscious client who underwent head surgery. Which site would be best used to monitor body temperature? 1 Skin 2 Oral 3 Axilla 4 Rectal

Rectal Although the oral route is the most common route for monitoring body temperature, clients who are unconscious should have their temperatures monitored rectally. Skin temperature may be impaired due to diaphoresis; this measurement may not reliable. The axilla temperature may underestimate the core temperature.

SKIN COLOR VARIATIONS: Pallor (decrease in color)

Reduced amt of oxyhemoglobin. Reduced visibility of oxyhemoglobin resulting from decreased bl flow. Causes: Anemia, Shock Assessment locations: Face, conjunctivae, nail beds, skin, nail beds

When meeting the unique preoperative teaching needs of an older adult, the nurse plans a teaching program based on the principle that learning:

Requires continued reinforcement

A client undergoes a bowel resection. When assessing the client 4 hours postoperatively, the nurse identifies which finding as an early sign of shock?

Restlessness

A *non inflammatory* eye disorder resulting from changes in *retinal blood vessels*. It is a *leading cause of blindness*

Retinopathy

A client is admitted to the hospital with severe diarrhea, abdominal cramps, and vomiting after eating. These symptoms have lasted 5 days. Upon further assessment, the primary healthcare provider finds that the symptoms occurred after the client ate eggs, salad dressings, and sandwich fillings. Which food borne disease would be suspected in this client?

Salmonellosis

A nurse educator is presenting information about the nursing process to a class of nursing students. What definition of the nursing process should be included in the presentation?

Sequence of steps used to meet the client's needs

A client is admitted with severe diarrhea that resulted in hypokalemia. The nurse should monitor for what clinical manifestations of the electrolyte deficiency?

Tachycardia & muscle weakness

A client has received instructions to take 650 mg aspirin (ASA) every 6 hours as needed for arthritic pain. What should the nurse include in the client's medication teaching?

Take the aspirin with meals or a snack, Do not chew enteric-coated tablets, Report persistent abdominal pain

A client comes to the clinic complaining of a productive cough with copious yellow sputum, fever, and chills for the past two days. The first thing the nurse should do when caring for this client is to:

Take the temperature

A graduate nurse is preparing to apply to the State Board of Nursing for licensure to practice as a licensed practical nurse. What group primarily is protected under the regulations of the practice of nursing?

The public

A nurse is caring for a client with diarrhea. The nurse anticipates a decrease in which clinical indicator? 1 Pulse rate 2 Tissue turgor 3 Specific gravity 4 Body temperature

Tissue turgor Skin elasticity will decrease because of a decrease in interstitial fluid. The pulse rate will increase to oxygenate the body's cells. Specific gravity will increase because of the greater concentration of waste particles in the decreased amount of urine. The temperature will increase, not decrease.

The nurse is caring for a client whose forehead feels warm to the touch. The nurse uses a thermometer and obtains the client's temperature. What is the nurse doing?

Validation The nurse is validating the presence of fever in the client.

the process of gathering more assessment data. It involves clarifying vague or unclear data.

Validation is

Phlebitis is an inflammation of

the inner layer of the vein. The findings for this include redness, tenderness, pain, and warmth along the course of the vein starting at the access site.

A nurse is educating a patient about Maslow's hierarchy of needs by citing examples. Which examples mentioned by the nurse belong to the third level of needs? .

"A client is depressed because his/her spouse has passed away." "A client wants to reconnect with old friends after being diagnosed with cancer." "A client never goes to family gatherings because he/she is not accepted by family members."

What is a nurse's most appropriate response, based on current research, when asked about spanking as a disciplinary technique?

"Spanking is strongly suggestive of negative role behavior."

While receiving a preoperative enema, a client starts to cry and says, "I'm sorry you have to do this messy thing for me." What is the nurse's best response?

"You seem upset."

An 11-year-old child has gained weight. The mother tells the nurse that she is concerned that her child, who loves sports, may become obese. What is the most appropriate response by the nurse? 1 Suggesting an increase in activity 2 Encouraging a decreased caloric intake 3 Explaining that this is expected during preadolescence 4 Discussing the influence of genetics on the child's weight gain

3 Explaining that this is expected during preadolescence There may be weight gain caused by the influence of hormones before the growth spurt. Most 10- to 12-year-old children can eat an adult-sized meal without becoming obese, especially if they are active. Before advising increased activity, the nurse should assess the child's current activity level. An adequate caloric intake is needed for the growth spurt that will occur during adolescence. Family eating patterns appear to have more effect on weight than do genetics.

Which Korotkoff sound represents the diastolic pressure in children? 1 First 2 Second 3 Fourth 4 Fifth

3 Fourth The fourth Korotkoff sound represents the diastolic pressure in children. The first Korotkoff sound represents the systolic pressure. The fifth Korotkoff sound represents the diastolic pressure in adults and adolescents. A blowing or swishing sound occurs in the second Korotkoff sound.

The average birth weight of a newborn is

3.2 to 3.4 kg.

An 18-month-old toddler with celiac disease is to be started on a gluten-free diet. The nurse teaches the parents to identify foods that should be avoided. Which food named by the parents as one to be avoided indicates to the nurse that the teaching has been successful? 1 Steamed rice 2 Mashed corn 3 Fresh applesauce 4 Grilled frankfurter

4 Grilled frankfurter Frankfurters usually have grain fillers; parents should read labels and, unless they are sure of the ingredients, refrain from feeding the food to their child. Steamed rice does not contain gluten. Mashed corn does not contain gluten. Applesauce does not contain gluten.

The nurse instructs a client that, in addition to building bones and teeth, calcium is also important for:

Blood clotting

SKIN COLOR VARIATIONS: Yellow-Orange jaundace

Causes: Liver disease, destruction of RBC's Condtion: increased deposit of bilirubin in tissues Assessment location: sclera, mucus membrane

Which site should be monitored for a pulse to assess the status of circulation to the foot? Select all that apply.

Dorsalis pedis artery Posterior tibial artery

What nursing actions best promote communication when obtaining a nursing history?

Establishing eye contact, paraphrasing the client's message, using broad, open-ended statements

intaocular structural damage resulting from elevated intraocular pressure. Obstruction of the outflow of aqueous humor causes this. Without treatment, leads to blindness

Glaucoma

intraocular structural damage resulting from elevated intraocular pressure.

Glaucoma is

Behind knee in popliteal fossa. Site used to assess stauts of circ in lower leg

Popliteal pulse

Which drug requires the nurse to monitor the client for signs of hyperkalemia?

Spironolactone (Aldactone)

When caring for a client with varicella and disseminated herpes zoster, the nurse should implement which types of precautions?

Standard, airborne, contact

Which assessment should the nurse exclude when dealing with a client with receptive and expressive aphasia?

Test the mental status by asking for feedback from the client Receptive and expressive aphasia are the two types of aphasia. A client with receptive is unable to understand written or verbal speech. A client with expressive aphasia understands written and verbal speech but cannot write or speak appropriately. A client with aphasia may not have the mental ability to give feedback; asking for feedback is ineffective.

A nursing student gives different examples representing the components of human personality, according to Sigmund Freud. Which examples mentioned by the nursing student represent the component ego?

"A client wants to eat junk food but does not order it because the dietician is keeping track of his or her weight." "A client with insomnia wants to take an extra sleeping pill but refrains from doing so because the caregivers are present." "A client is attracted to the nurse in charge but does not make any moves because there are other clients in the same ward."

A nurse teaches a client about various measures to protect against food-borne illness. Which statement by the client indicates a need for further teaching?

"I won't eat any leftovers in my refrigerator after they've been there for 5 days."

The nurse administers a pneumococcal vaccine to a 70-year-old client. The client asks "Will I have to get this every year like I do with the flu shot?" How should the nurse respond? 1 "You need to receive the pneumococcal vaccine every other year." 2 "The pneumococcal vaccine should be received in early autumn every year." 3 "You should get the flu and pneumococcal vaccines at your annual physical examination." 4 "It is unnecessary to have any follow-up injections of the pneumococcal vaccine after this dose."

"It is unnecessary to have any follow-up injections of the pneumococcal vaccine after this dose." The Centers for Disease Control and Prevention recommend that adults be immunized with pneumococcal vaccine at age 65 years or older with a single dose of the vaccine; if the pneumococcal vaccine was received before 65 years of age or if there is the highest risk of fatal pneumococcal infection, revaccination should occur 5 years after the initial vaccination. The pneumococcal vaccine should not be administered every 2 years. The pneumococcal vaccine should not be administered annually.

A nurse teaches a client about wearing thigh-high anti-embolism elastic stockings. What would be appropriate to include in the instructions?

"You will need to apply them in the morning before you lower your legs from the bed to the floor."

An older adult client who is receiving chemotherapy for cancer has severe nausea and vomiting and becomes dehydrated. The client is admitted to the hospital for rehydration therapy. Which nursing actions have specific gerontologic implications the nurse must consider? Select all that apply.

- Assessment of skin turgor - Administration of antiemetic drugs - Replacement of fluid and electrolytes

A client presents with bilateral leg pain and cramping in the lower extremities. The client has a history of cardiovascular disease, diabetes, and varicose veins. To guide the assessment of the pain and cramping, the nurse should include which question when completing the initial assessment? 1 "Does walking for long periods of time increase your pain?" 2 "Does standing without moving decrease your pain?" 3 "Have you had your potassium level checked recently?" 4 "Have you had any broken bones in your lower extremities?"

1. "Does walking for long periods of time increase your pain?" Clients with a medical history of heart disease, hypertension, phlebitis, diabetes, or varicose veins often develop vascular-related complications. The nurse should recognize that the relationship of symptoms to exercise will clarify whether the presenting problem is vascular or musculoskeletal. Pain caused by a vascular condition tends to increase with activity. Musculoskeletal pain is not usually relieved when exercise ends. Low potassium levels can cause cramping in the lower extremities; however, given the client's health history, vascular insufficiency should be suspected. Previously healed broken bones do not cause cramping and pain. Test-Taking Tip: Do not spend too much time on one question, because it can compromise your overall performance. There is no deduction for incorrect answers, so you are not penalized for guessing. You cannot leave an answer blank; therefore, guess. Go for it! Remember: You do not have to get all the questions correct to pass.

The palmar surface of the hand and finger pads Are more sensitive than the fingertips. These should be used to determine .

1. position 2. texture 3. size 4. consistency 5. masses 6. fluid 7. crepitus

The nurse performs a respiratory assessment and auscultates breath sounds that are high pitched, creaking, and accentuated on expiration. Which term best describes the findings? 1 Rhonchi 2 Wheezes 3 Pleural friction rub 4 Bronchovesicular

2 Wheezes Wheezes are one of the most common breath sounds assessed and auscultated in clients with asthma and chronic obstructive pulmonary disease (COPD). Wheezes are produced as air flows through narrowed passageways. Rhonchi are coarse, rattling sounds similar to snoring and are usually caused by secretions in the bronchial airways. A pleural friction rub is an abrasive sound made by two acutely inflamed serous surfaces rubbing together during the respiratory cycle. Bronchovesicular sounds are intermediate between bronchial (upper) and vesicular (lower) breath sounds; they are normal when heard between the first and second intercostal spaces anteriorly and posteriorly between scapulae. Test-Taking Tip: Be alert for details. Details provided in the stem of the item, such as behavioral changes or clinical changes (or both) within a certain time period, can provide a clue to the most appropriate response or, in some cases, responses.

What is a nurse's most appropriate response, based on current research, when asked about spanking as a disciplinary technique? 1 "Effectiveness depends on the child's age." 2 "Spanking is strongly suggestive of negative role behavior." 3 "Spanking may be the only option when no other technique works." 4 "Research studies have shown it to be an effective disciplinary technique."

2 "Spanking is strongly suggestive of negative role behavior." Research suggests that children who are spanked tend to use aggressive behavior; as they grow older they learn their own behavior through their parents' behavior. Age is not significant in terms of the effectiveness of spanking. Research studies contradict the assertion that spanking is an effective disciplinary technique.

A 78-year-old client who has hypertension is beginning treatment with furosemide. Considering the client's age, what should the nurse teach the client to do? 1 Limit fluids at bedtime. 2 Change positions slowly. 3 Take the medication between meals. 4 Assess the skin for breakdown daily.

2 Change positions slowly. With aging there is a decreased vasomotor response and diminished elasticity of blood vessels, which therefore do not respond quickly to changes from horizontal to vertical; orthostatic hypotension may occur. Changing positions slowly allows the body to adjust, which prevents dizziness and loss of balance. Usual fluid intake patterns can be maintained. Furosemide should be taken with meals to prevent gastric irritation. It is best to take it in the morning rather than at night so that sleep is not interrupted with the need to void. There is no link between furosemide and skin breakdown.

The nurse asks the client to shrug the shoulders and to turn the head against passive resistance. Which cranial nerve is involved in this action? 1 Cranial nerve II 2 Cranial nerve XI 3 Cranial nerve VI 4 Cranial nerve VII

2 Cranial nerve XI Cranial nerve XI (the spinal accessory nerve) is the motor nerve that coordinates the movement of head and shoulders. Cranial nerve II (optic nerve) is a sensory nerve for visual acuity. Cranial nerve VI (abducens nerve) is a motor nerve that coordinates the lateral movement of eyeballs. Cranial nerve VII or (auditory nerve) is a sensory nerve which coordinates the hearing sense.

While assessing a pediatric client, an ophthalmologist notices that the child is unable to focus on an object with both eyes simultaneously. Which other findings in the client confirms the diagnosis as strabismus? Select all that apply. 1 Impaired near vision 2 Crossed appearance of eyes 3 Elevated intraocular pressure 4 Impaired extraocular muscles 5 Degeneration of central retina

2 Crossed appearance of eyes 4 Impaired extraocular muscles Strabismus is a congenital condition in which both eyes do not focus on an object simultaneously, resulting in a crossed appearance of the eyes. This condition is caused by impaired extraocular muscles. Impaired near vision is associated with hyperopia or presbyopia. Elevated intraocular pressure results in glaucoma. Macular degeneration is caused by degeneration of the central retina.

What clinical finding does a nurse anticipate when admitting a client with an extracellular fluid volume excess? 1 Rapid, thready pulse 2 Distended jugular veins 3 Elevated hematocrit level 4 Increased serum sodium level

2 Distended jugular veins Because of fluid overload in the intravascular space, the neck veins become visibly distended. Rapid, thready pulse and elevated hematocrit level occur with a fluid deficit. If sodium causes fluid retention, its concentration is unchanged; if fluid is retained independently of sodium, its concentration is decreased.

A client with osteoporosis is encouraged to drink milk. The client refuses the milk, explaining that it causes gas and bloating. Which food should the nurse suggest that is rich in calcium and digested easily by clients who do not tolerate milk? 1 Eggs 2 Yogurt 3 Potatoes 4 Applesauce

2 Yogurt Yogurt, which contains calcium, is digested more easily than milk because it contains the enzyme lactase, which breaks down milk sugar. Yogurt contains approximately 274 to 415 mg of calcium for an 8 oz (237 mL) container depending on how it is prepared. Eggs contain approximately 22 mg of calcium. One potato contains approximately 7 to 20 mg of calcium depending on how it is prepared. Eight ounces of applesauce contain approximately 3 mg of calcium.

A nurse in the ambulatory preoperative unit identifies that a client is more anxious than most clients. What is the nurse's best intervention? 1 Attempt to identify the client's concerns. 2 Reassure the client that the surgery is routine. 3 Report the client's anxiety to the health care provider. 4 Provide privacy by pulling the curtain around the client.

1 Attempt to identify the client's concerns. The nurse should assess the situation before planning an intervention. Reassuring the client that the surgery is routine minimizes concerns and cuts off communication. Reporting the client's anxiety to the health care provider is premature; more information is needed. The nurse needs more information; pulling the curtain may make the client feel isolated, which may increase anxiety.

Which nursing activities are examples of primary prevention?

Assisting with immunization programs, facilitating a program about smoking cessation

involved in downward and inward eye movements.

The trochlear nerve is *CNIV*(4)

During history taking, a client reports experiencing black, tarry stools. The nurse recognizes that this may be an indication of

Upper gastrointestinal bleeding

Which error will result in false high diastolic readings while measuring a client's blood pressure during a physical examination?

Inflating the cuff too slowly Inflating or deflating the cuff too slowly will yield false high diastolic readings.

While undergoing a soapsuds enema, the client reports abdominal cramping. What action should the nurse take?

Lower the height of the enema bag

A client with Addison's disease is receiving cortisone therapy. The nurse expects what clinical indicators if the client abruptly stops the medication?

Tachypnea, hypotension

What should the nurse include in dietary teaching for a client with a colostomy?

The diet should be adjusted to include foods that result in manageable stools

While assessing a client's hair, a nurse notices that the client has head lice. The nurse teaches the client about hair hygiene and lice control. Which statements made by the client indicates an understanding of the teaching?

"I should use a dilute vinegar solution to loosen the nits." "I should use a shampoo treatment once every 24 hours." "I will clean my comb in ammonia water."

A nurse reinforces teaching a client about Coumadin (warfarin) and concludes that the teaching is effective when the client states, "I must not drink:

Cranberry juice

What should the nurse do initially when obtaining consent for surgery?

Determine whether the client's knowledge level is sufficient to give consent

A nurse is preparing to administer an oil-retention enema and understands that it works primarily by:

Lubricating the sigmoid colon and rectum

What are the clinical indicators that a nurse expects when an intravenous (IV) line has infiltrated?

Pallor, edema, decreased flow rate

A nurse assesses for hypocalcemia in a postoperative client. One of the initial signs that might be present is 1 Headache. 2 Pallor. 3 Paresthesias. 4 Blurred vision.

3 Paresthesias. Normally, calcium ions block the movement of sodium into cells. When calcium is low, this allows sodium to move freely into cells, creating increased excitability of the nervous system. Initial symptoms are paresthesias. This can lead to tetany if untreated. Headache, pallor, and blurred vision are not signs of hypocalcemia.

A client who experienced extensive burns is receiving intravenous fluids to replace fluid loss. The nurse should monitor for which initial sign of fluid overload? 1 Crackles in the lungs 2 Decreased heart rate 3 Decreased blood pressure 4 Cyanosis

1 Crackles in the lungs Crackles, or rales, in the lungs are an early sign of pulmonary congestion and edema caused by fluid overload. Clients with fluid overload will usually demonstrate an increased heart rate and increased blood pressure. A decreased heart rate and decreased blood pressure and cyanosis in a client with fluid overload would be very late and fatal signs.

A nurse is assessing a client with a history of marijuana use. Which long-term effects are associated with marijuana? Select all that apply. 1 Lung cancer 2 Emphysema 3 Heart disease 4 Laryngeal disorder 5 Stroke 6 Chronic nasal irritation

1 Lung cancer 2 Emphysema 3 Heart disease Lung cancer, emphysema, and heart disease are outcomes that may occur due to marijuana use. Laryngeal disorders, stroke, and chronic nasal irritation are associated with the abuse of cocaine but are not associated with marijuana use.

An older adult is found to have a thin white ring around the margin of the iris. What condition does this denote? 1 Cataract 2 Arcus senilis 3 Conjunctivitis 4 Macular degeneration

2 Arcus senilis In older adults, the iris becomes faded and a thin white ring (known as arcus senilis) appears around the margin of the iris. A cataract is a condition involving increased opacity of the lens that blocks light rays from entering the eye. The presence of redness indicates allergic or infectious conjunctivitis. Macular degeneration is marked by a blurring of central vision caused by progressive degeneration of the center of the retina.

Norm heart rate for Newborns (0 to 1 month old):

70 to 190 beats per minute.

A nurse suspects that a client has poison ivy. Assessment findings reveal vesicles on the arms and legs. A vesicle can be described as:

A lesion filled with serous fluid

A hospitalized client experiences a fall after climbing over the bed's side rails. Upon reviewing the client's medical record, the nurse discovers that restraints had been prescribed but were not in place at the time of the fall. What information should the nurse include in the follow-up incident report?

A listing of facts related to the incident as witnessed by the nurse

The nurse is performing a skin assessment. Which illustration may represent a tumor?

A solid mass that extends deep through the subcutaneous tissue may indicate a skin tumor called an epithelioma (as seen in the first figure). A palpable, circumscribed, solid elevation in the skin indicates the formation of a papule (as seen in the second figure). An elevated solid mass that is deeper and firmer than a papule indicates the formation of a nodule (as seen in the third figure). A circumscribed elevation of the skin that is similar to a vesicle but filled with pus indicates a pustule (as seen in the fourth figure).

The nurse cares for an unconscious client who underwent head surgery. Which site would be best used to monitor body temperature?

Although the oral route is the most common route for monitoring body temperature, clients who are unconscious should have their temperatures monitored *rectally*.

4th to 5th intercostal space at L midclavicular line. Point of Maximal impulse (PMI)

Apical pulse

supine position

Areas assessed: head, neck, ant thorax, heart, lungs, breast, axilla, abdomen, extremities, pulses Rationale: The most relaxed position. Provides easy access to pulse site Limitations: if pt becomes short of breath easily raise head of the bed

A female client explains to the nurse that she sleeps until noon every day and takes frequent naps during the rest of the day. What should the nurse do initially?

Arrange a referral for a thorough medical evaluation

A nurse is taking care of a client who has severe back pain as a result of a work injury. What nursing considerations should be made when determining the client's plan of care?

Ask the client what is the client's acceptable level of pain & administer the pain medications regularly around the clock

The nurse is preparing to teach a client about self-injection of insulin. Which action by the nurse will increase the effectiveness of the teaching session?

Assess clients barriers to learning the new technique

Which client is suspected of having hypertension based on the given data? Client, cardiac output, peripheral resistance, hematocrit 1. A, decreased, normal, decreased 2. B, increased, increased, increased 3. C, decreased, normal, normal 4. D, normal, increased, normal

B The blood pressure (BP) in a client rises when the client's cardiac output, peripheral resistance, and hematocrit are increased. Because all of these parameters are increased in client B, then that client is suspected to have hypertension. The BP falls when cardiac output is decreased. So, clients A and C may be at risk of hypotension. Client D's cardiac output may not be at risk of hypertension.

A nurse is teaching a parenting class. What should the nurse suggest about managing the behavior of a young school-age child?

Be consistent about established rules.

Space b/t biceps & triceps at antecubital fossa. Site used to assess circulation to low arm and ausclutate BP.

Brachial Pulse

Along medial edge of sternocleidomastoid muscle in neck. Easily accessible site used durig physologocal shock or cardaic arrest weh other sites are not palpable

Carotid pulse

An increased opacity of the lens, which blocks light rays from entering the eye.sometimes develop slowly and progressively after 35 or sudden trauma. One of the most common eye disorders. Most adults 65 & older have some evidence of visual impairment from this disorder

Cataracts

can occur if intravenous solutions are infused too rapidly or in great amounts.

Circulatory overload

The findings of four clients who underwent eye examinations are given below. Which client is suspected to have sustained injury to the cranial nerve III?

Client C - crossed eyes

Which client is at the highest medical risk of coronary heart disease and hypertension? 1. Client A - height (cm) 180 weight (kg) 70 2. Client B - height (cm) 185 weight (kg) 95 3. Client C - height (cm) 152 weight (kg) 56 4 Client D - height (cm) 145 weight (kg) 67

D A body mass index (BMI) higher than 30 is considered obesity and puts the client at a higher medical risk of coronary heart disease, some cancers, and hypertension. Client D (who is 145 cm tall and weighs 67 kg) has a BMI of 31.9, which indicates obesity. This can lead to coronary heart disease and hypertension. Client A has a BMI of 21.6, which indicates a normal weight. Client B has a BMI of 27.77, which indicates that the client is overweight but not obese. Client C, with a BMI of 24.24, is considered as having a normal weight.

According to Kübler-Ross, during which stage of grieving are individuals with serious health problems most likely to seek other medical opinions?

Denial

After abdominal surgery a client reports pain. What action should the nurse take first?

Determine the characteristics of the pain.

The nurse assesses a client's pulse and documents the strength of the pulse as 3+. The nurse understands that this pulse can be characterized as what?

Full

The nurse is caring for a client who is on a low carbohydrate diet. With this diet, there is decreased glucose available for energy, and fat is metabolized for energy resulting in an increased production of which substance in the urine?

Ketones

A client with hemiplegia is staring blankly at the wall and reports feeling like half a person. What is the most appropriate initial nursing action?

Offer to spend more time with the client.

Sims position

Rectum & Vag Flexion of hip and knee improves exposure of rectal area Joint deformities hinder patient's ability to ben hip and knee

Edema is classified as pitting if .

the application of pressure on the edematous site will leave an indentation for some time

knee-chest position

the patient is lying face down with the hips bent so that the knees and chest rest on the table This position is embarrassing and uncomfortable. Provides maximum exposure of rectal area. Used to examine the rectum

An accumulation of edematous fluid will result in

the separation of skin and underlying vasculature.

An adolescent that had an inguinal hernia repair is being prepared for discharge home. The nurse provides instructions about resumption of physical activities. Which statement by the adolescent indicates that the client understands the instructions?

"I can't perform any weightlifting for at least 3 weeks."

A nursing student is recording the radial pulse rate in a client with dysrhythmias and documented a radial pulse of 80 beats per minute. The registered nurse reassesses the client and notices a pulse deficit of 15. What is the client's apical pulse? 1 95 2 85 3 75 4 65

1 95 A nursing student is recording the radial pulse rate in a client with dysrhythmias and documented a radial pulse of 80 beats per minute. The registered nurse reassesses the client and notices a pulse deficit of 15. What is the client's apical pulse?

The nurse is performing a skin assessment of a client. Which findings in the client may indicate a risk of skin cancer? Select all that apply. 1 Lesion 2 Lumps 3 Rashes 4 Bruising 5 Dryness

1 Lesion 2 Lumps 3 Rashes Lesions on the skin that take a long time to heal may indicate skin cancer. Lumps and rashes on the skin are characteristics of skin cancer. Bruising may indicate a bleeding disorder or injury. Dryness of the skin may be due to excessive bathing and use of harsh soaps.

While caring for a client with heat stroke, the nurse measured the temperature and noted it as 39ºC. What is this temperature in Fahrenheit? Record your answer using one decimal place. _____ºF

102.2 Celsius is converted to Fahrenheit by multiplying the Celsius reading by 9/5 and adding the product to 32. In this case, the calculation is: (9/5)(39) + 32 = 102.2.

The registered nurse is teaching a nursing student about bulimia nervosa in adolescents. Which statement made by the nursing student indicates effective learning? 1 "The client claims to feel fat despite being underweight." 2 "The client experiences recurrent episodes of binge eating." 3 "The client exhibits intense fear of gaining weight although underweight." 4 "The client refuses to maintain body weight over a minimal ideal body weight."

2 "The client experiences recurrent episodes of binge eating." Bulimia nervosa is an eating disorder in which the client has an obsessive desire to lose weight. In this condition, bouts of extreme overeating are followed by fasting or self-induced vomiting. A recurrent episode of binge eating is an indicator of bulimia nervosa. A client claims to feel fat despite being underweight may have anorexia nervosa. Other assessment findings of anorexia nervosa include an intense fear of gaining weight despite being underweight and a refusal to maintain a body weight over a minimal ideal body weight.

Which landmark is correct for a nurse to use when auscultating the mitral valve? 1 Left fifth intercostal space, midaxillary line 2 Left fifth intercostal space, midclavicular line 3 Left second intercostal space, sternal border 4 Left fifth intercostal space, sternal border

2 Left fifth intercostal space, midclavicular line The correct landmark for auscultating the mitral valve (apical pulse) is found at the left fifth intercostal space (ICS) in the midclavicular line. Auscultation at the fifth ICS in the midaxillary line would yield breath sounds of the lateral lung field. Auscultation at the left second ICS at the sternal border is best to hear the pulmonic valve, and at the left fifth ICS at the sternal border for the tricuspid valve.

What clinical finding indicates to the nurse that a client may have hypokalemia?

Abdominal distention

A client with rheumatoid arthritis does not want the prescribed cortisone and informs the nurse. Later, the nurse attempts to administer cortisone. When the client asks what the medication is, the nurse gives an evasive answer. The client takes the medication and later discovers that it was cortisone. The client states an intent to sue. What factors in this situation must be considered in a legal action?

Clients have a right to refuse treatment, nurses are required to answer clients truthfully, the health care provider should have been notified

Health promotion efforts within the health care system should include efforts related to secondary prevention. Which activities reflect secondary prevention interventions in relation to health promotion?

Encouraging regular dental checkups, teaching the procedure for breast self-examination

A client complains of difficulty breathing. The nurse auscultates wheezing in the anterior bilateral upper lobes. What could be the possible reason for this sound?

High velocity airflow through an obstructed airway

Common eye/vision problem: farsightedness, a refractive error in which rays of light enter the eye and *focus behind the retina*. Persons are able to clearly see distinct objects but not close objects.

Hyperopia

Which internal variable influences health beliefs and practices?

Intellectual background is an internal factor that affects the client's health beliefs and practices.

A nurse is assessing an older adult male client. Which clinical findings are expected responses to the aging process? Select all that apply

Slowed neurologic responses Forgetfulness about recent events Reduced ability to maintain an erection

While assessing a client who sustained a road traffic accident, a nurse notices that the client is unable to clench his teeth. Which cranial nerve might have been affected?

The *trigeminal nerve* provides sensory innervation to the facial skin and motor innervation to the muscles of the jaw. A client with a damaged trigeminal nerve will be unable to clench his teeth. *CNV*

Sims position

lying on left side with right knee drawn up and with left arm drawn behind, parallel to the back

A client complains of rapid, involuntary movement of the eyes after a minor eye injury. A nurse assesses the client and finds that it is a disorder of the cranial nerves. Which condition does the nurse suspect?

*Nystagmus* is a condition defined by rapid, involuntary, rhythmical oscillation of the eyes. This condition is caused by local injury to the eye muscles and supporting structures.

Which statement made by the nurse indicates that the client interview is coming to a close? 1 "I have just one more question for you." 2 "I hope you are comfortable and not in pain." 3 "I would like to spend some time to understand your concerns." 4 "I assure you that information I gather now will be confidential."

1 "I have just one more question for you." The nurse should give the client a clue that the interview is drawing to a close. The nurse can do this by letting the client know that after one more question the interview will be over. The nurse sets the stage for the interview by ensuring that the client is comfortable and not in pain. The nurse begins the interview by stating that he or she would like to spend some time to understand the client's health concerns. The nurse informs the client at the beginning of the interview that the information shared by the client is confidential. Test-Taking Tip: Avoid looking for an answer pattern or code. There may be times when four or five consecutive questions have the same letter or number for the correct answer.

A registered nurse is teaching a nursing student about precautions to be taken for physical examination of a client. Which statements made by the nursing student indicate effective learning? Select all that apply. 1 "I should examine the client in noise-free areas." 2 "I should use latex gloves during the physical examination." 3 "I should perform a physical examination in a cool room." 4 "I should leave a combative client alone during a physical examination." 5 "I should wear eye shields while examining a client with excessive drainage."

1 "I should examine the client in noise-free areas." 5 "I should wear eye shields while examining a client with excessive drainage." Clients should be examined in noise-free areas to prevent interruptions. Wearing eye shields while examining a client with excessive drainage helps to reduce contamination. Latex gloves should be used with caution because they may cause allergy in clients who are allergic to latex. A physical examination should be performed in a warm room to minimize discomfort. Combative clients should never be left alone during physical examinations. Test-Taking Tip: Be alert for details about what you are being asked to do. In this Question Type, you are asked to select all options that apply to a given situation or client. All options likely relate to the situation, but only some of the options may relate directly to the situation.

A registered nurse is teaching a nursing student about when a client with high blood pressure should follow up with the primary healthcare provider. Which statement made by the nursing student indicates effective learning? 1 "I will advise a client with a blood pressure of 130/80 mm Hg to follow up in a year." 2 "I will advise a client with a blood pressure of 110/70 mm Hg to follow up in a year." 3 "I will advise a client with a blood pressure of 150/90 mm Hg to follow up in a month." 4 "I will advise a client with a blood pressure of 185/115 mm Hg to follow up in a month."

1 "I will advise a client with a blood pressure of 130/80 mm Hg to follow up in a year." A client with prehypertension tends to have a blood pressure (BP) between 120/80 and 139/89 mm Hg. These clients should be rechecked in a year. Clients with BP less than 120/80 mm Hg are considered normal. These clients should be rechecked in two years. Clients with stage 1 hypertension have a BP between 140/90 and 159/99 mm Hg. These clients should be rechecked in two months to confirm stage 1 hypertension. Clients with stage 2 hypertension have a BP greater than 160/100 mm Hg. These clients should be rechecked in one month. If a client's BP is greater than 180/110 mm Hg, then he or she should be treated immediately or within 1 week.

While assessing a client's range of motion, the nurse explains adduction to the nursing student. Which statement made by the nursing student indicates effective learning? 1 "I will ask the client to move his or her arm towards the body." 2 "I will ask the client to bend his or her limb by decreasing the angle." 3 "I will ask the client to move his or her hand so that the ventral surface faces downward." 4 "I will ask the client to move his or her head beyond its normal resting extended position."

1 "I will ask the client to move his or her arm towards the body." Adduction is moving the arm towards the body. Assessing the range of motion by bending the limb and decreasing the angle indicates flexion. Moving the hand by facing the ventral surface downwards indicates pronation. The movement of the head beyond the normal resting extended position indicates hyperextension.

The nurse is assessing a client who arrived at the healthcare facility for an appointment. Which action by the nurse will be beneficial during the interview? 1 Asking about the client's current concerns 2 Ensuring the interview follows a strict agenda 3 Asking questions that promote short responses by the client 4 Telling the client what he or she should expect from the visit

1 Asking about the client's current concerns The nurse should begin the interview by gathering information about the client's current concerns to encourage the client to express his or her chief problems. The nurse then sets an agenda for the interview. However, the nurse must remember that the best interview focuses on the client and not the nurse's agenda. The nurse must ask open-ended questions that allow the client to describe his or her concerns more clearly. The nurse should ask the client to describe his or her healthcare expectations to help the client understand that the nurse is genuinely interested in the client's health. Test-Taking Tip: You have at least a 25 percent chance of selecting the correct response in multiple-choice items. If you are uncertain about a question, eliminate the choices that you believe are wrong and then call on your knowledge, skills, and abilities to choose from the remaining responses.

The nurse is assessing a client who reports breathlessness. Which activity best ensures that the nurse obtains accurate and complete data to prevent a nursing diagnostic error? 1 Assess the client's lungs. 2 Assess the client for pain. 3 Obtain details of smoking habits. 4 Ask about the onset of breathlessness.

1 Assess the client's lungs. The nurse should assess the client's lungs to gather objective data that will support subjective data provided by the client. The nurse can obtain objective data for this client by auscultating for lung sounds, assessing the respiratory rate, and measuring the client's chest excursion. The nurse should review the data for accuracy and completeness before grouping the data into clusters. The nurse may also assess the client for pain, which is subjective data. The client may provide details about smoking habits, which is also subjective data. It is important for the nurse to identify what causes breathlessness; however, the client's statement is subjective data. All subjective data must be supported by measurable objective data.

Which clients should be considered for assessing the carotid pulse? Select all that apply. 1 Client with cardiac arrest 2 Client indicated for Allen test 3 Client under physiologic shock 4 Client with impaired circulation to foot 5 Client with impaired circulation to hand

1 Client with cardiac arrest 3 Client under physiologic shock Carotid pulse is indicated in clients with physiologic shock or cardiac arrest when other sites are not palpable in the client. Assessment of the ulnar pulse is indicated in clients requiring an Allen test. Assessment of posterior tibial pulse and dorsalis pedis pulse is indicated in clients with impaired circulation to the feet. Assessment of the radial and ulnar pulse is indicated in clients with impaired circulation to the hands. Test-Taking Tip: Be alert for details about what you are being asked to do. In this Question Type, you are asked to select all options that apply to a given situation or patient. All options likely relate to the situation, but only some of the options may relate directly to the situation.

Which client is suspected to have an increased risk of hyperlipidemia? Select all that apply. 1 Client with corneal arcus 2 Client with periorbital edema 3 Client with decreased skin turgor 4 Client with paleness of conjunctivae 5 Client with yellow lipid lesions on eyelids

1 Client with corneal arcus 5 Client with yellow lipid lesions on eyelids The presence of corneal arcus, which is the whitish opaque ring around the junction of the cornea and sclera, indicates that the client has hyperlipidemia. Yellow lipid lesions on the eyelids refer to xanthelasma, which indicates a client has hyperlipidemia. The presence of periorbital edema indicates the client may have kidney disease. Decreased skin turgor may be due to dehydration. Paleness of the conjunctivae indicates anemia.

Which actions by the nurse help set the stage for a patient-centered interview during the first visit after admission to the healthcare facility? Select all that apply. 1 Close the door after entering the room. 2 Greet the client using his or her last name. 3 Open the curtains to allow plenty of light in the room. 4 Introduce oneself with a smile and explain the reason for the visit. 5 Obtain an authorization from the client after the interview.

1 Close the door after entering the room. 2 Greet the client using his or her last name. 4 Introduce oneself with a smile and explain the reason for the visit. The nurse should maintain the client's privacy by closing the door after entering the room. The nurse should maintain the dignity of the client by greeting the client using his or her last name. Smiling is a positive sign of warmth and immediacy when first establishing the nurse-client relationship. The nurse should explain his or her role in the providing care for the client. The nurse should ensure the room is adequately lit, comfortable, and soothing for the client. The nurse need not open the curtains to allow plenty of light in the room. The Health Insurance Portability and Accountability Act (HIPAA) requires the nurse to obtain an authorization from the client before collecting personal health data.

The nurse is assessing a young client who presents with recurrent gastrointestinal disorders. On further assessment, the nurse learns that the client is experiencing job-related pressure. What is the most important nursing intervention for this client? 1 Educate the client on managing stress. 2 Teach the client to maintain a balanced diet. 3 Instruct the client to have regular health checkups. 4 Ask the client to use sunscreen when working outdoors.

1 Educate the client on managing stress. The client is experiencing job-related pressure, so the nurse should educate the client about managing stress as it is a lifestyle risk factor. Stress threatens both mental health and physical well-being. Stress is associated with illnesses such as heart disease, cancer, and gastrointestinal disorders. The nurse teaches the client to maintain a balanced diet as a primary preventive care to promote health. The nurse should instruct the client to have regular health checkups as a primary preventive measure. The nurse should ask the client to use sunscreen when working outdoors to avoid excess sun exposure and prevent skin cancer.

A client with recent history of head trauma is at risk of orthostatic hypotension. Which assessment findings would help to diagnose the condition? Select all that apply. 1 Fainting 2 Headache 3 Weakness 4 Light headedness 5 Shortness of breath

1 Fainting 3 Weakness 5 Light headedness Head trauma may cause blood loss. Clients with recent blood loss are at risk of orthostatic hypotension. While obtaining the orthostatic measurements, the nurse should check for fainting, light-headedness, and weakness. Headaches and shortness of breath are the symptoms of hypertension. Topics

A client suffers hypoxia and a resultant increase in deoxygenated hemoglobin in the blood. What are the best sites to assess this condition? Select all that apply. 1 Lips 2 Sclera 3 Mouth 4 Sacrum 5 Nail beds 6 Shoulders

1 Lips 3 Mouth 5 Nail beds Prolonged hypoxia resulting in increased amounts of deoxygenated blood causes cyanosis, which can be best evaluated in lips, mouth, nail beds, and skin (in extreme conditions). Sclera is the site of assessment for jaundice, while shoulders are assessed to confirm the condition of erythema.

After performing an optical assessment on a client, a primary healthcare provider notices impaired near vision. Which other finding in this client confirms the diagnosis as presbyopia? 1 Loss of elasticity of the lens 2 Increased opacity of the lens 3 Elevated intraocular pressure 4 Noninflammatory changes in eyes

1 Loss of elasticity of the lens Presbyopia is defined as impaired near vision caused by a loss of elasticity of the lens. This condition is reported in middle-aged and older adults. Increased opacity of the lens is seen in cataracts. Elevated intraocular pressure is associated with glaucoma. Retinopathy causes noninflammatory eye changes. Test-Taking Tip: You have at least a 25% chance of selecting the correct response in multiple-choice items. If you are uncertain about a question, eliminate the choices that you believe are wrong and then call on your knowledge, skills, and abilities to choose from the remaining responses.

Which features distinguish nursing diagnoses from medical diagnoses? Select all that apply. 1 Nursing diagnoses involve the client when possible. 2 Nursing diagnoses are based on results of diagnostic tests and procedures. 3 Nursing diagnoses are the identification of a disease condition in the client. 4 Nursing diagnoses involve the sorting of health problems within the nursing domain. 5 Nursing diagnoses involve clinical judgment about the client's response to health problems

1 Nursing diagnoses involve the client when possible. 4 Nursing diagnoses involve the sorting of health problems within the nursing domain. 5 Nursing diagnoses involve clinical judgment about the client's response to health problems Establishing a nursing diagnosis is the second step in the nursing process. It is unique and involves the client's participation in the process. Nursing diagnoses classify health problems to be treated primarily by nurses. The nurse reviews the client assessment, sees cues and patterns in the data, and identifies the client's specific health care problems. The nursing diagnosis is a clinical judgment about the client's actual or potential health problems that the nurse is licensed to treat. A medical diagnosis is based on results of diagnostic tests and procedures, whereas a nursing diagnosis is based on the results of the nursing assessment. A medical diagnosis identifies a disease condition in the client.

While caring for a postoperative client, the nurse observed a pulse deficit during physical assessment. Which pulses are used to assess the pulse deficit? 1 Radial and apical pulse 2 Apical and carotid pulse 3 Radial and brachial pulse 4 Apical and temporal pulse

1 Radial and apical pulse Pulse deficit may be associated with an abnormal rhythm. Pulse deficit is the difference between the radial and apical pulse. The carotid pulse is measured when a client's condition worsens suddenly. The brachial pulse is used to measure blood pressure. The temporal pulse is used to assess the pulse in children.

The findings of four clients who underwent eye examinations are given below. Which client is suspected to have sustained injury to the cranial nerve III? Client, eye finding 1. drooping eye 2. nearsightedness 3. cross-eye 4. protruding eyes

1. Drooping eyes Injury to the third cranial nerve may result in edema or impairment of the third cranial nerve. This results in the abnormal drooping of the eyelids, a condition called ptosis. Myopia is nearsightedness, a refractive error in which rays of light enter the eye and focus in front of the retina. Cross-eyes result from strabismus, which results from neuromuscular injury or congenital anomaly. Protruding eyes (exophthalmoses) is indicative of hyperthyroidism.

If the drainage output is less than the amount of irrigation solution infused, or if the patient complains of pain during bladder irrigation, do the following:

1. Examine the drainage tubing for clots, sediment, and kinks. 2. Inspect the urine for the presence of or increase in blood clots and sediment. 3. Evaluate the patient for pain and bladder distention. 4. Notify the health care provider.

A registered nurse is supervising a student nurse while assessing a 70-year-old client who is receiving aminoglycoside therapy. Which statement about the client's condition requires correction? 1 "The client may have deterioration of the cochlea." 2 "The client may have thinning of the tympanic membrane." 3 "The client may have an inability to hear high-frequency sounds." 4 "The client may have an inability to differentiate between consonants."

2 "The client may have thinning of the tympanic membrane." An older adult who is on aminoglycoside antibiotic therapy is at a high risk of developing ototoxicity. The client with ototoxicity may have thickening of the tympanic membrane, but not thinning of the tympanic membrane. Deterioration of the cochlea may cause older adults to gradually lose hearing. They may experience an inability to hear high-frequency sounds and differentiate between consonants. Test-Taking Tip: Identify option components as correct or incorrect. This may help you identify a wrong answer.

A nurse is reviewing the laboratory report of a client with kidney problems. When ammonia is excreted by healthy kidneys, what mechanism usually is maintained? 1 Osmotic pressure of the blood 2 Acid-base balance of the body 3 Low bacterial levels in the urine 4 Normal red blood cell production

2 Acid-base balance of the body The excreted ammonia combines with hydrogen ions in the glomerular filtrate to form ammonium ions, which are excreted from the body. This mechanism helps rid the body of excess hydrogen, maintaining acid-base balance. Osmotic pressure of the blood and normal red blood cell production are not affected by excretion of ammonia. Ammonia is formed by the decomposition of bacteria in the urine; ammonia excretion is not related to the process and does not control bacterial levels.

A client reports to the nurse sleeping until noon every day and taking frequent naps during the rest of the day. What should the nurse do initially? 1 Encourage the client to exercise during the day. 2 Arrange a referral for a thorough medical evaluation. 3 Explain that this behavior is an attempt to avoid facing daily responsibilities. 4 Identify that the client is describing clinical findings associated with narcolepsy.

2 Arrange a referral for a thorough medical evaluation. This behavior is a sign of hypersomnia, and the client needs a medical assessment; it is commonly caused by central nervous system damage or certain kidney, liver, or metabolic disorders. Exercise is appropriate for a client experiencing insomnia, not hypersomnia. This behavior is a sign of hypersomnia, and medical causes should be ruled out before attributing it to a psychogenic cause. Narcolepsy consists of recurrent sudden waves of overwhelming sleepiness that occur during the day, even during activities such as eating or conversing.

A client was admitted to a surgical unit in an unconscious state due to head trauma. Which site would be most appropriate to obtain the client's temperature? 1 Oral 2 Axilla 3 Temporal artery 4 Tympanic membrane

2 Axilla The axilla would be the most appropriate site to obtain a temperature measurement in a client who is unconscious due to head trauma. The oral route is not accessible when the client is unconscious. Because the client is in a surgical unit, his or her head may be covered. Therefore, obtaining a temperature measurement through the temporal artery or tympanic membrane may not be possible. Test-Taking Tip: Be alert for details. Details provided in the stem of the item, such as behavioral changes or clinical changes (or both) within a certain time period, can provide a clue to the most appropriate response or, in some cases, responses.

A client complains of sudden muscle weakness during times of anger or laughter that may occur at any time during the day. Which condition should be suspected in this client? 1 Insomnia 2 Cataplexy 3 Narcolepsy 4 Sleep apnea

2 Cataplexy Cataplexy is a condition in which muscle weakness occurs suddenly during times of intense emotion, such as anger, sadness, or laughter. A cataplexic attack may occur at any time during the day. Insomnia is a condition in which a person has chronic difficulty falling asleep. A person with narcolepsy also experiences frequent awakenings from sleep, short periods of sleep or nonrestorative sleep, or some combination thereof. Narcolepsy is a dysfunction of the mechanisms that regulate sleeping and waking states. Sleep apnea is a disorder characterized by a lack of airflow through the nose and mouth for periods of 10 seconds or longer during sleep.

While assessing a client, a nurse finds that the ratio of the anteroposterior diameter and transverse diameter of the chest is 1:1. What is indicated by this finding? Select all that apply. 1 Client has lordosis. 2 Client is an older adult. 3 Client has osteoporosis. 4 Client has a history of smoking. 5 Client has chronic lung disease.

2 Client is an older adult. 4 Client has a history of smoking. 5 Client has chronic lung disease. The 1:1 ratio of the anteroposterior diameter and transverse diameter of the chest indicates a barrel-shaped chest. This is a characteristic feature in an older adult who smokes and has chronic lung disease. In lordosis, there is an increase in lumbar curvature. Osteoporosis is a systemic skeletal condition in which there is a decreased bone mass and deterioration of bone tissue.

An 85-year-old client has just been admitted to a nursing home. When designing a plan of care for this older adult, the nurse recalls what expected sensory losses associated with aging? Select all that apply. 1 Difficulty in swallowing 2 Diminished sensation of pain 3 Heightened response to stimuli 4 Impaired hearing of high frequency sounds 5 Increased ability to tolerate environmental heat

2 Diminished sensation of pain 4 Impaired hearing of high frequency sounds Because of aging of the nervous system, an older adult has a diminished sensation of pain and may be unaware of a serious illness, thermal extremes, or excessive pressure. As people age they experience atrophy of the organ of Corti and cochlear neurons, loss of the sensory hair cells, and degeneration of the stria vascularis, which affects an older person's ability to perceive high frequency sounds. An interference with swallowing is a motor loss, not a sensory loss, and it is not an expected response to aging. There is a decreased, not heightened, response to stimuli in older adults. There is a decreased, not increased, ability to physiologically adjust to extremes in environmental temperature.

The community nurse is assessing an elderly client who lives alone at home. The nurse finds that the client refrains from physical activity for fear of falling when walking. Which interventions by the nurse are most beneficial to promote a healthy lifestyle? Select all that apply. 1 Instruct the client to apply bed side rails. 2 Encourage the client to wear nonskid shoes. 3 Suggest that the client use an assistive device. 4 Ask the client to install hand rails in the bathroom. 5 Help the client rearrange furniture in the house.

2 Encourage the client to wear nonskid shoes. 3 Suggest that the client use an assistive device. 5 Help the client rearrange furniture in the house. The nurse should encourage the client to wear nonskid shoes that will provide a firm grip while walking and help reduce the chance of falls. The nurse should suggest that the client use an assistive device such as a cane or walker for support while walking. The nurse should make environmental changes by helping the client rearrange the furniture in the house. This will help reduce the incidence of falls within the house. These interventions reduce the fear of falling and encourage the client to participate in physical activity indoors and outdoors. The bed side rails protect the client from falling from the bed. The hand rails in the bathroom assist provide support while using the bathroom.

The nurse teaching a health awareness class identifies which situation as being the highest risk factor for the development of a deep vein thrombosis (DVT)? 1 Pregnancy 2 Inactivity 3 Aerobic exercise 4 Tight clothing

2 Inactivity A DVT, or thrombus, may form as a result of venous stasis. It may lodge in a vein and can cause venous occlusion. Inactivity is a major cause of venous stasis leading to DVT. Pregnancy and tight clothing are also risk factors for DVT secondary to inactivity. Aerobic exercise is not a risk factor for DVT. Test-Taking Tip: You have at least a 25% chance of selecting the correct response in multiple-choice items. If you are uncertain about a question, eliminate the choices that you believe are wrong and then call on your knowledge, skills, and abilities to choose from the remaining responses.

A nurse is assessing an 89-year-old client with a history of severe congenital spinal deformity. Which condition would most likely describe the nurse's finding? 1 Lordosis 2 Kyphosis 3 Presbycusis 4 Osteoporosis

2 Kyphosis Kyphosis is an increase in the curvature of the thoracic spine and may result from a congenital abnormality. Lordosis, also known as swayback, is an increased lumbar curvature and may not be a congenital abnormality. Presbycusis is the loss of acuity for high-frequency tones and is not related to the spine. Osteoporosis is a condition in which the bones become brittle and fragile from the loss of tissue and bone mass.

A client with hyperthyroidism has been treated with radioactive iodine (131I) to destroy overactive thyroid gland cells. To reduce radiation exposure, the nurse's principles for providing care should be based on: 1 Wearing a lead-shield apron at all times 2 Limiting distance and time spent with the client 3 Wearing a radiation meter to measure exposure 4 Remaining at least 6 feet away from the client at all times

2 Limiting distance and time spent with the client When caring for clients who are radioactive, the three most important concepts for reducing radiation exposure are to limit exposure time, increase distance, and use shielding. In this situation, time and distance provide the best reduction in radiation exposure. Wearing a lead-shield apron will help prevent radiation exposure, but time and distance are the first priorities. A radiation meter measures exposure but does nothing to protect caretakers. Remaining at least 6 feet away from the client at all times is not a practical approach.

A registered nurse (RN) is instructed to assess the body temperature of a neonate. Which site for placing the thermometer is contraindicated in these clients? 1 Axilla 2 Oral cavity 3 Temporal artery 4 Tympanic membrane

2 Oral cavity The oral cavity is the preferred site for temperature measurement in adult clients. This site is contraindicated for neonates and unconscious or uncooperative clients. The axilla is a safe site for placing a thermometer in neonates. The temporal artery is indicated for rapid temperature measurement. This site is indicated for premature infants, newborns, and children. The tympanic membrane is indicated in newborns to reduce infant handling and heat loss.

How does the nurse identify an illness as chronic? Select all that apply. 1 The illness is reversible and often severe. 2 The illness persists for longer than six months. 3 The client may develop life threatening relapse. 4 The symptoms are intense and appear abruptly. 5 The illness affects the functioning of one or more systems.

2 The illness persists for longer than six months. 3 The client may develop life threatening relapse. 5 The illness affects the functioning of one or more systems. A chronic illness usually lasts longer than six months. The client with chronic illness often fluctuates between maximal functioning and serious health relapses that may be life threatening. The illness affects the functioning of one or more systems. A chronic illness is irreversible, whereas an acute illness is reversible and often much more severe than a chronic illness. The client with acute illness develops intense symptoms that appear abruptly and often subside after a relatively short period.

Which nursing intervention is employed to encourage the client to fully reveal the nature of their health problem? 1 The nurse takes down notes while the client is talking. 2 The nurse leans forward attentively during the discussion. 3 The nurse refrains from pausing enough after each question. 4 The nurse asks questions that can be answered as "yes" or "no."

2 The nurse leans forward attentively during the discussion. The nurse leans forward attentively to show awareness, attention, and immediacy during discussion with the client. This encourages the client to fully reveal the nature of his or her health problem. The client may feel that the nurse is too busy to pay attention if the nurse takes down notes during the discussion. If the nurse hurries with the assessment, the client is likely to understand that the nurse is busy or not interested. The nurse should ask open-ended questions that encourage the client to give detailed information about his or her health. Closed-ended questions that can be answered with "yes" or "no" often bring the conversation to a close.

During a physical assessment, a client was diagnosed with increased temperature due to an increased basal metabolic rate (BMR). Which hormonal imbalances may the client have? Select all that apply. 1 Cortisol 2 Thyroid 3 Estrogen 4 Testosterone 5 Progesterone

2 Thyroid 4 Testosterone Body temperature is assessed during physical assessment. An increased basal metabolism rate increases the body temperature. Hormonal imbalances may alter the basal metabolic rate (BMR). Testosterone regulates the BMR in males. Thyroid hormone regulates the BMR of the body. Increases in the levels of these hormones may increase the BMR, which may in turn raise body temperature. Cortisol regulates blood glucose levels. Estrogen and progesterone are female hormones that do not regulate the BMR. Test-Taking Tip: Be alert for details about what you are being asked to do. In this Question Type, you are asked to select all options that apply to a given situation or client. All options likely relate to the situation, but only some of the options may relate directly to the situation.

While performing a physical assessment of a female client, the nurse positions the client in Sims' position. Which body system will be assessed in this position? Select all that apply. 1 Heart 2 Vagina 3 Rectum 4 Female genitalia 5 Musculoskeletal system

2 Vagina 3 Rectum Sims' position is indicated to examine vagina and rectum. Lithotomy to check female genitalia. Lateral recumbent position will aid in detecting murmurs of the heart. Prone position is indicated while assessing the musculoskeletal system.

A nurse is performing physical assessment of four female clients who came for a general checkup. Which client is most at risk of developing breast cancer? age, family history of breast cancer, children, age of menopause 1. 60, yes, 2, 45 2.60, yes, none, 50 3.60, no, none, 50 4. 60, no, 2, 45

2. Women over 40 years of age with a personal or family history of breast cancer, late-age menopause (after age 50), who have not had children or who conceived after the age of 30 years, or women with excessive oral contraceptives use are at risk of developing breast cancer. Client B has all the criteria that increase the risk of developing breast cancer, such as age over 60 years, positive family history, no children, and menopause at a later age. Therefore she is at the highest risk of developing breast cancer compared to clients A, C, and D. Client A has children and therefore has a relatively lower risk of developing breast cancer. Client C has negative family history and therefore has a relatively lower risk of developing breast cancer. Client D has children and also has a negative family history, and therefore has a relatively lower risk of developing breast cancer.

A nursing student has prepared pulse assessment plans for several clients. Which client's assessment plan is correct and will yield effective results? 1. Site - Ulnar - Location - Ulnar side of the forearm at the wrist - Assessment criteria - cardiac arrest when other sites are not palpable 2. Carotid - Along the medial edge of the sternocleidomastoid muscle in the neck. 3. Dorsalis pedis - along the top of the foot - status of the circulation to the foot 4. Posterior Tibial - Above the medial malleolus - status of circulation to the foot

3 The dorsalis pedis is located along the top of the foot. This site is used to assess the status of circulation in the foot. The ulnar site, found on the ulnar side of the forearm at the wrist, is used to assess the status of circulation to the hand and to perform the Allen test. The carotid site is found along the medial edge of the sternocleidomastoid muscle of the neck. It is easily accessible in times of physiological shock or cardiac arrest when other sites are not palpable. The posterior tibial site is found below (not above) the medial malleolus. It is used to assess the status of circulation in the foot.

A client develops an allergic reaction when a student nurse is performing a physical assessment. Which statement made by the student nurse in response to this incident indicates the need for further teaching? 1 "Type I immune response to latex has an immediate onset." 2 "Type I immune reaction to latex leads to release of IgE antibodies." 3 "The client's first exposure to latex will cause a type IV allergic reaction." 4 "Type IV immune response to latex occurs after 12 to 48 hours after exposure."

3 "The client's first exposure to latex will cause a type IV allergic reaction." Both type I and type IV hypersensitive reactions require prior exposure to cause an immune response in a subsequent exposure. The most immediate immune response is a type I reaction, in which the body produces IgE antibodies against the allergen. A type IV immune response occurs 12 to 48 hours after the exposure to the allergen and is referred to as a delayed hypersensitivity response.

While assessing a 7-month-old infant, the nurse advises the mother to avoid regular cow's milk. Which of these are valid reasons for the suggestion? Select all that apply. 1 Cow's milk is not tolerated by infants. 2 Cow's milk is a potential source of botulism toxin. 3 Cow's milk increases the risk of milk product allergies. 4 Cow's milk is a poor source of iron and vitamins C and E. 5 Cow's milk is too concentrated for an infant's kidneys to manage.

3 Cow's milk increases the risk of milk product allergies. 4 Cow's milk is a poor source of iron and vitamins C and E. 5 Cow's milk is too concentrated for an infant's kidneys to manage. Regular cow's milk is avoided in infants during the first year of life because it is too concentrated for an infant's kidneys to manage. It also increases the risk of milk product allergies and is also a poor source of iron and vitamins C and E. Honey and corn syrup are potential sources of botulism toxin. Most infants are not allergic to or intolerant of cow's milk. Test-Taking Tip: Be alert for details about what you are being asked to do. In this Question Type, you are asked to select all options that apply to a given situation or client. All options are likely related to the situation, but only some of the options may be related directly to the situation.

The nurse is providing postoperative care to a client who had a submucosal resection (SMR) for a deviated septum. The nurse should monitor for what complication associated with this type of surgery? 1 Occipital headache 2 Periorbital crepitus 3 Expectoration of blood 4 Changes in vocalization

3 Expectoration of blood After an SMR, hemorrhage from the area should be suspected if the client is swallowing frequently or expelling blood with saliva. A headache in the back of the head is not a complication of a submucosal resection. Crepitus is caused by leakage of air into tissue spaces; it is not an expected complication of SMR. The nerves and structures involved with speech are not within the operative area. However, the sound of the voice is altered temporarily by the presence of nasal packing and edema. Test-Taking Tip: Start by reading each of the answer options carefully. Usually at least one of them will be clearly wrong. Eliminate this one from consideration. Now you have reduced the number of response choices by one and improved the odds. Continue to analyze the options. If you can eliminate one more choice in a four-option question, you have reduced the odds to 50/50. While you are eliminating the wrong choices, recall often occurs. One of the options may serve as a trigger that causes you to remember what a few seconds ago had seemed completely forgotten.

A mother is worried about the sudden behavioral changes in her child. The child has suddenly developed a fear of certain people and places. The child's school performance is declining rapidly, and the child has developed poor relationships with his or her peers. After assessing the physical findings of the child, the nurse suspects child abuse. Which physical findings might have led the nurse to this suspicion? 1 Sunken eyes and loss of weight 2 Uncommunicative and uninteractive with others 3 Foreign bodies in the rectum, urethra, or vagina 4 Strangulation marks on neck from rope burns or bruises

3 Foreign bodies in the rectum, urethra, or vagina One of the physical findings that may be required to confirm child abuse is the presence of foreign bodies in the rectum, urethra, or vagina. Weight loss and sunken eyes may be a physical finding for older adult abuse. When the abuse is related to an intimate partner, the nurse may observe strangulation marks on the neck from rope burns or bruises. Staying isolated and not communicating with others are behavioral findings that may be related to older adult abuse.

A primary nurse receives prescriptions for a newly admitted client and has difficulty reading the health care provider's writing. Who should the nurse ask for clarification of this prescription? 1 Nurse practitioner 2 House health care provider that is on-call 3 Health care provider who wrote the prescription 4 Nurse manager familiar with the health care provider's writing

3 Health care provider who wrote the prescription The health care provider who wrote the prescription should be called for clarification. The nurse is liable and responsible if the prescription is misinterpreted. Only the health care provider who wrote an undecipherable prescription can correctly clarify the prescription.

The nurse is performing an assessment of the client's reproductive system. Which finding of the past medical history indicates the client is at risk of cervical cancer? 1 Vaginal discharge 2 Ovarian dysfunction 3 Human papilloma virus infection 4 Hematuria and urinary incontinence

3 Human papilloma virus infection A human papilloma virus (HPV) infection increases the risk of cervical cancer. The presence of vaginal discharge may indicate a sexually transmitted disease. A history of ovarian dysfunction may increase the risk of ovarian cancer. The presence of hematuria and urinary incontinence may indicate urinary problems associated with gynecological disorders. Test-Taking Tip: Cervical cancer is the cancer arising from the cervix. Recall the risk factors of cervical cancer and choose the correct answer.

A nurse is caring for a client who underwent cardiac catheterization. The client's skin was found to be blanched, and there was formation of edema of 15.2 cm (1-6 inches) at the site of catheterization. Upon further assessment, the skin was found to be cool, and the client complains of tenderness. Which condition does the nurse expect? 1 Phlebitis 2 Infection 3 Infiltration 4 Circulatory overload

3 Infiltration The client with blanched skin, edema of 15.2 cm, cool temperature, and pain at the site of catheterization has symptoms of grade 2 infiltration. Phlebitis is an inflammation of the inner layer of the vein. The findings for this include redness, tenderness, pain, and warmth along the course of the vein starting at the access site. If there is infection, there will be findings that include redness, heat, swelling at catheter-skin entry point, and possible purulent drainage. Circulatory overload can occur if intravenous solutions are infused too rapidly or in great amounts.

Which finding is inferred from a grade 4 intensity of heart murmurs? 1 Thrill is easily palpable 2 Quiet and clearly audible thrill 3 Loud murmur associated with thrill 4 Moderately loud murmur without thrill

3 Loud murmur associated with thrill Grade 4 indicates loud murmurs with an associated thrill. A thrill is a fine vibration that is felt by palpation. A grade 5 intensity is characterized by an easily palpable thrill. A grade 2 intensity is characterized by quiet and clearly audible murmurs. A moderately loud murmur without a thrill is noted as grade 3.

The nurse is caring for a client with a family history of diabetes mellitus. The client has been following a diet regimen recommended by the dietician and walking for 45 minutes daily for the past eight months. How should the nurse document the client's stage based on the transtheoretical model of health behavior change? 1 Action 2 Preparation 3 Maintenance 4 Contemplation

3 Maintenance The client is in the maintenance stage of human behavior change. During this stage, the client has managed to incorporate the changes in to the lifestyle. This stage begins six months after the action has started and continues indefinitely. The action stage lasts for six months from the time the client has incorporated the changes in to the lifestyle. During the preparation stage, the client begins to realize that the advantages of the change outweigh the disadvantages. The client starts making small changes in preparation for major changes the following month. During the contemplation stage, the client is still considering whether to incorporate changes in the next six months.

A client complains of rapid, involuntary movement of the eyes after a minor eye injury. A nurse assesses the client and finds that it is a disorder of the cranial nerves. Which condition does the nurse suspect? 1 Cataract 2 Glaucoma 3 Nystagmus 4 Strabismus

3 Nystagmus Nystagmus is a condition defined by rapid, involuntary, rhythmical oscillation of the eyes. This condition is caused by local injury to the eye muscles and supporting structures. A cataract is a condition in which the opacity of the lens will be increased; this disorder is commonly related to age. Glaucoma is intraocular structural damage resulting from elevated intraocular pressure. Strabismus is a congenital condition in which both eyes do not focus on an object simultaneously. In this condition, the eyes appear crossed.

The nurse discovers several palpable elevated masses on a client's arms. Which term most accurately describes the assessment findings? 1 Erosions 2 Macules 3 Papules 4 Vesicles

3 Papules Papules are superficial and elevated up to 0.5 cm. Nodules and tumors are masses similar to papules, but are elevated more than 0.5 cm and may infiltrate deeper into tissues. Erosions are characterized as loss of the epidermis layer; macules are nonpalpable, flat changes in skin color less than 1 cm in diameter; and vesicles are usually transparent, filled with serous fluid, and are a blisterlike elevation. Test-Taking Tip: Practicing a few relaxation techniques may prove helpful on the day of an examination. Relaxation techniques such as deep breathing, imagery, head rolling, shoulder shrugging, rotating and stretching of the neck, leg lifts, and heel lifts with feet flat on the floor can effectively reduce tension while causing little or no distraction to those around you. It is recommended that you practice one or two of these techniques intermittently to avoid becoming tense. The more anxious and tense you become, the longer it will take you to relax.

The nurse has just arrived in the unit for her shift at the healthcare facility. There are two new clients admitted to the unit. What should the nurse do first to collect the first set of information about the clients assigned to his or her care? 1 Meet the clients' family. 2 Read the clients' medical reports. 3 Participate in the bedside rounds. 4 Visit the clients and introduce self.

3 Participate in the bedside rounds. The nurse should participate in bedside rounds with the healthcare team from the previous shift. The nurse who is completing care for one shift prepares the change-of-shift report to communicate client details to the nurse in the next shift. These bedside rounds provide patient-centered care as the nurse shares information about the client's condition, status of problems, and treatment plan for the next shift. The nurse can meet the client's family after obtaining firsthand information from the nurse completing the shift. The nurses review the client's medical reports and discuss treatment plans for the next shift after completing the bedside rounds. The nurse may meet the client during bedside rounds or after obtaining the handover report.

The nurse documents the data gathered during the assessment in a client's medical record. What should the nurse do to ensure that the data is meaningful to other healthcare providers? 1 Record subjective information in own words. 2 Form judgments through written communication. 3 Record objective information using accurate terminology. 4 Compare data from the physical examination with client behavior.

3 Record objective information using accurate terminology. The nurse should document all objective information using accurate terminology. The nurse should pay attention to the facts and report findings exactly as seen, felt, or smelled. If the information is not specific, another healthcare provider reading the data gets only general impressions. The nurse should record subjective information in quotations, exactly as described by the client. The nurse should refrain from generalizing or forming judgments during documentation. This information is used to form nursing diagnoses, which must be factual and accurate. During validation, the nurse compares data from the physical examination with client behavior.

An 8-year-old child admitted to the hospital 2 days ago tells the nurse, "I'm too sick to feed myself." The nurse recognizes that this statement is most likely indicative of: 1 Rebellion 2 Loneliness 3 Regression 4 Immaturity

3 Regression Regression is the retreat to a past level of behavior as a way of minimizing stress or controlling anxiety. Increased dependence, such as being fed by another person, is a form of regression. The child's statement does not indicate rebellion against those who are providing care. Although loneliness may be a factor, it is not the key factor in the child's statement. The child's statement reflects not immaturity but rather an assumption of the sick role to reduce stress.

When nurses are conducting health assessment interviews with older clients, what step should be included? 1 Leave a written questionnaire for clients to complete at their leisure. 2 Ask family members rather than the client to supply the necessary information. 3 Spend time in several short sessions to elicit more complete information from the clients. 4 Keep referring to previous questions to ascertain that the information given by clients is correct.

3 Spend time in several short sessions to elicit more complete information from the clients. Spending time in several short sessions reduces client fatigue and compensates for a shortened attention span, which is common in the older adult. The questionnaire may never be completed if it is left for the client to complete at their leisure. Asking family members rather than the client to supply the necessary information is degrading to the client; the client should be asked initially and, if necessary, family can be asked to fill in details later. Constantly referring to previous questions may be overwhelming and create feelings of anger and resentment.

While performing a physical assessment in a client, the registered nurse (RN) notices reddish linear streaks in the nail bed. Which systemic condition can the registered nurse (RN) suspect in the client based on these assessment findings? 1 Syphilis 2 Iron deficiency anemia 3 Subacute bacterial endocarditis 4 Chronic obstructive pulmonary disease

3 Subacute bacterial endocarditis Red or brown linear streaks in the nail bed are caused by minor trauma to nails, subacute bacterial endocarditis, or trichinosis and are called splinter hemorrhages. Conditions such as syphilis and iron deficiency anemia cause concavely curved nails, called koilonychia. Heart and lung abnormalities such as chronic obstructive pulmonary disease cause clubbing of the nail beds.

A registered nurse teaches a nursing student about routines followed during a physical examination to help ensure that important findings are not missed. Which statement by the nursing student indicates ineffective learning? 1 "I'll compare the two sides of the body for symmetry." 2 "I'll record quick notes during the examination to avoid delays." 3 "I'll perform painful procedures at the beginning of the examination." 4 "I'll record assessments in specific terms in the electronic or paper record."

3"I'll perform painful procedures at the beginning of the examination." Any painful procedures should be performed at the end of the examination. The two sides of the body should be compared for symmetry, because some asymmetries are abnormal. Recording quick notes during the examination will help prevent delays during the examination. More extensive notes may be completed at the end of the examination. Assessments should be recorded in specific terms in the electronic or paper record. This standard form allows information to be recorded in the same sequence in which it is gathered.

While caring for different clients with respiratory problems, the nurse assesses the different alterations in the breathing pattern. Which client's assessment findings indicate Cheyne-Stokes respiration? Client, breathing pattern, beats per min 1. regular and abnormally snow, 11 2. regular but abnormally rapid, 25 3. Irregular, alternating apnea and hyperventilation, 12 4. Abnormally shallow for two breaths followed by apnea, 30

3. Irregular, alternating apnea and hyperventilation, 12 In Cheyne-Stokes respiration, a client's breathing pattern is characterized by progressively deeper and faster breathing, that is, hyperventilation followed by apnea. Client 3 exhibits this type of respiration. Client 1's breathing pattern indicates bradypnea, while client 2 exhibits tachypnea. Client 4 is exhibiting Biot's respirations. Test-Taking Tip: Chart/exhibit items present a situation and ask a question. A variety of objective and subjective information is presented about the client in formats such as the medical record (e.g., laboratory test results, results of diagnostic procedures, progress notes, health care provider orders, medication administration record, health history), physical assessment data, and assistant/patient interactions. After analyzing the information presented, the test taker answers the question. These questions usually reflect the analyzing level of cognitive thinking.

A registered nurse is teaching a nursing student about skin assessment. Which statement made by the nursing student indicates the need for further teaching? 1 "Skin assessments are best performed in daylight." 2 "Skin assessments performed at cool room temperatures can result in cyanosis." 3 "Skin assessment performed at warm room temperatures can result in vasodilatation." 4 "In the absence of sunlight, skin assessments are performed best with other sources of light instead of fluorescent light."

4 "In the absence of sunlight, skin assessments are performed best with other sources of light instead of fluorescent light." Though skin assessments are best conducted in daylight, in the absence of sunlight, they are best performed in fluorescent lighting. Skin exposure during skin assessments in cool room temperature can result in cyanosis. Skin exposure during skin assessments made in warm room temperature can result in vasodilatation.

A nurse is assessing several clients. Which client will require parenteral nutrition? 1 A client with brain neoplasm 2 A client with anorexia nervosa 3 A client with inflammatory bowel disease 4 A client with severe malabsorption disorder

4 A client with severe malabsorption disorder A client with severe malabsorption disorder requires parenteral nutrition. Clients with a brain neoplasm, anorexia nervosa, or inflammatory bowel disease will require enteral nutrition.

A nurse assesses a client with dry and brittle hair, flaky skin, a beefy-red tongue and bleeding gums. The nurse recognizes that these clinical manifestations are most likely a result of: 1 A food allergy. 2 Noncompliance with medications. 3 Side effects from medications. 4 A nutritional deficiency.

4 A nutritional deficiency. All of the signs listed are classic for a poor nutritional state lacking in basic nutrients such as vitamins and protein. A specific food allergy or medication is not described, therefore there is not enough information to assume the signs and symptoms are related to either.

While assessing an immobilized client, the nurse notes that the client has shortened muscles over a joint, preventing full extension. What is this condition known as? 1 Osteoarthritis 2 Osteoporosis 3 Muscle atrophy 4 Contracture

4 Contracture Immobilized clients are at high risk for the development of contractures. Contractures are characterized by permanent shortening of the muscle covering a joint. Osteoarthritis is a disease process of the weight-bearing joints caused by wear and tear. Osteoporosis is a metabolic disease process in which the bones lose calcium. Muscle atrophy is a wasting and/or decrease in the strength and size of muscles because of a lack of physical activity or a neurologic or musculoskeletal disorder.

An elderly client is admitted to the healthcare facility following a stroke. What should the nurse do when the client's relative who arrived much later asks to see the client's health record? 1 Confirm the client's relationship first. 2 Ask the client's primary healthcare provider. 3 Inform the nurse manager and show the records. 4 Explain that medical health records are confidential.

4 Explain that medical health records are confidential. The Health Insurance Portability and Accountability Act (HIPAA) sets the standards for the protection of the client's health information. The nurse must explain that medical health records are confidential. The healthcare team must be aware of the organization's policies for reviewing a client's medical record for assessment. The nurse need not confirm the client's relationship because the client's medical records are confidential and cannot be shared with anyone unless authorized by the client. The primary healthcare provider cannot authorize the nurse to show the medical records. The nurse cannot inform the nurse manager and show the medical record to persons not involved in direct client care. Healthcare providers share information with reasonable safeguards within the healthcare team for the purpose of providing client care. Test-Taking Tip: The nurse must follow the standards set by The Health Insurance Portability and Accountability Act.

A client complains of difficulty breathing. The nurse auscultates wheezing in the anterior bilateral upper lobes. What could be the possible reason for this sound? 1 Inflammation of the pleura 2 Muscular spasms in the larger airways 3 Sudden reinflation of groups of alveoli 4 High velocity airflow through an obstructed airway

4 High velocity airflow through an obstructed airway Wheezing is a high-pitched sound that may be caused by a high velocity airflow through an obstructed or narrowed airway. Inflammation of the pleura may produce pleural friction rubs. Muscular spasms in larger airways or any new growth causing turbulence may produce rhonchi, which is a loud and low-pitched sound. Sudden reinflation of groups of alveoli may produce crackling sounds.

A client has a fever spike that is combined with normal temperature levels. The client's body temperature returns to a normal body temperature at least once a day. Which type of fever can be assessed in the client? 1 Sustained 2 Relapsing 3 Remittent 4 Intermittent

4 Intermittent An intermittent fever is characterized by fever spikes interspersed with normal temperatures. In this type of fever, the body temperature returns to normal at least once in 24 hours. In the case of sustained fever, there is a constant body temperature greater than 38ºC. In relapsing fever, there is an occurrence of periods of febrile episodes with acceptable temperature values. In remittent fever, the body temperature increases and decreases without returning to normal body temperature levels.

The nurse is caring for an African American client with renal failure. The client states that the illness is a punishment for sins. Which cultural health belief does the client communicate? 1 Yin/Yang balance 2 Biomedical belief 3 Determinism belief 4 Magicoreligious belief

4 Magicoreligious belief An African American client may have magicoreligious beliefs, which focuses on hexes or supernatural forces that cause illness. Such clients may believe that illness is a punishment for sins. The yin/yang belief system does not consider illness as a punishment. The biomedical belief system maintains that health and illness are related to physical and biochemical processes with disease being a breakdown of the processes. The belief of determinism focuses on outcomes that are externally preordained and cannot be changed. Test-Taking Tip: Identify option components as correct or incorrect. This may help you identify a wrong answer.

An infant with a diagnosis of heart failure is being given furosemide (Lasix) twice a day. Which laboratory value should the nurse report to the health care provider? 1 Sodium of 140 mEq/L 2 Ionized calcium of 2.35 mEq/L 3 Chloride of 102 mEq/L 4 Potassium of 3.0 mEq/L

4 Potassium of 3.0 mEq/L Because furosemide is a potassium-losing diuretic, the potassium should be frequently checked. Normal potassium concentration in infants is 3.5 to 5.0 mEq/L. A potassium concentration of 3.0 mEq/L should be reported to the health care provider. The normal range for sodium is 139 to 146 mEq/L, ionized calcium is normally 2.24 to 2.46 mEq/L, and chloride is normally 98 to 106 mEq/L. The sodium, calcium, and chloride readings are normal.

A nurse must establish and maintain an airway in a client who has experienced a near-drowning in the ocean. For which potential danger should the nurse assess the client? 1 Alkalosis 2 Renal failure 3 Hypervolemia 4 Pulmonary edema

4 Pulmonary edema Additional fluid from surrounding tissues will be drawn into the lung because of the high osmotic pressure exerted by the salt content of the aspirated ocean water; this results in pulmonary edema. Hypoxia and acidosis may occur after a near-drowning, not alkalosis. Renal failure is not a sequela of near-drowning. Hypovolemia occurs because fluid is drawn into the lungs by the hypertonic saltwater.

While assessing a client who sustained a road traffic accident, a nurse notices that the client is unable to clench his teeth. Which cranial nerve might have been affected? 1 Facial nerve 2 Trochlear nerve 3 Abducens nerve 4 Trigeminal nerve

4 Trigeminal nerve The trigeminal nerve provides sensory innervation to the facial skin and motor innervation to the muscles of the jaw. A client with a damaged trigeminal nerve will be unable to clench his teeth. The facial nerve provides sensory and motor innervations for facial expressions. The trochlear nerve is involved in downward and inward eye movements. The abducens nerve helps in the eyeball's lateral movement.

A client has a heart rate of 72 beats/min and stroke volume of 70 mL. What is the client's cardiac output? Record your answer using a whole number. _____mL/min

5040 The volume of blood pumped by the heart in 1 minute is the cardiac output. Cardiac output is the product of the heart rate and the stroke volume of the ventricle. Therefore cardiac output in the client with a heart rate of 72 beats/min and stroke volume of 70 mL is 5040 mL/min: 72 × 70 = 5040.

a visual representation of the connection between the client's many health problems: The first step is to arrange all the cues into clusters that form patterns. This helps the nurse identify specific nursing diagnoses for the client. During the assessment stage, the nurse assesses the client and gathers information. This step is performed before preparing the concept map. After placing all cues into clusters, the nurse begins to identify patterns reflecting the client's problem. The concept map helps the nurse obtain a holistic view of the client's needs. The next step is to identify specific diagnoses so that appropriate nursing interventions can be provided.

A concept map is

While assessing a client who experienced an accident, the nurse found that the client is unable to move eyeballs laterally. Which nerve damage led to this condition in the client?

Abducens nerve

A nurse is caring for an older adult who is taking acetaminophen (Tylenol) for the relief of chronic pain. Which substance is mostimportant for the nurse to determine if the client is taking because it intensifies the mostserious adverse effect of acetaminophen?

Alcohol

Which client is suspected to have an increased risk of hyperlipidemia? Select all that apply. 1 Client with corneal arcus 2 Client with periorbital edema 3 Client with decreased skin turgor 4 Client with paleness of conjunctivae 5 Client with yellow lipid lesions on eyelids

Client with corneal arcus Client with yellow lipid lesions on eyelids The presence of corneal arcus, which is the whitish opaque ring around the junction of the cornea and sclera, indicates that the client has hyperlipidemia. Yellow lipid lesions on the eyelids refer to xanthelasma, which indicates a client has hyperlipidemia. The presence of periorbital edema indicates the client may have kidney disease. Decreased skin turgor may be due to dehydration. Paleness of the conjunctivae indicates anemia.

Which client is expected to be diagnosed with macular degeneration?

Client with loss of central vision Client B's loss of central vision is caused by macular degeneration. Impaired near vision in client A is due to presbyopia or hyperopia. Strabismus is a congenital condition in which both eyes do not focus on an object simultaneously; this results in the cross appearance of eyes, as seen in client C. Client D's inability to see distant objects is caused by myopia. Test-Taking Tip: Identifying content and what is being asked about that content is critical to your choosing the correct response.

What would cause a client to have concavely curved nails?

Conditions such as iron deficiency anemia and syphilis cause concave curvature of the nails, which is called koilonychia (spoon nails). Red or brown linear streaks in the nail bed are caused by minor trauma to nails, subacute endocarditis, or trichinosis. They are called splinter hemorrhages. Softening of the nail bed and enlargement of the finger tips with flattened nails are signs of clubbing of nails, which is seen in conditions of oxygen deficiency such as in heart or pulmonary diseases, cyanotic heart disease, and chronic obstructive pulmonary disease. Test-Taking Tip: Chart/exhibit items present a situation and ask a question. A variety of objective and subjective information is presented about the client in formats such as the medical record (e.g., laboratory test results, results of diagnostic procedures, progress notes, health care provider orders, medication administration record, health history), physical assessment data, and assistant/patient interactions. After analyzing the information presented, the test taker answers the question. These questions usually reflect the analyzing level of cognitive thinking.

The nurse is preparing discharge instructions for a client who has begun to demonstrate signs of early Alzheimer's dementia. The client lives alone. The client's adult children live nearby. According to the prescribed medication regimen the client is to take medications six times throughout the day. What is priority nursing intervention to assist the client with compliance with medication-taking?

Contact the primary healthcare provider and discuss the possibility of simplifying the medication regimen.

A client who experienced extensive burns is receiving intravenous fluids to replace fluid loss. The nurse should monitor for which initial sign of fluid overload? 1 Crackles in the lungs 2 Decreased heart rate 3 Decreased blood pressure 4 Cyanosis

Crackles in the lungs Crackles, or rales, in the lungs are an early sign of pulmonary congestion and edema caused by fluid overload. Clients with fluid overload will usually demonstrate an increased heart rate and increased blood pressure. A decreased heart rate and decreased blood pressure and cyanosis in a client with fluid overload would be very late and fatal signs.

Arrange the steps taken by a nurse while assessing the visual level of a client in sequential order. Direct the client to stand or sit 60 cm away from eye level Close the opposite eye to superimpose the field of vision Ask the client to close his or her left or right eye gently and look directly at the nurse's opposite eye Ask the client to report when he or she is able to see the finger Move a finger equidistant between the nurse and the client outside the field of vision

Direct the client to stand or sit 60 cm away from eye level Ask the client to close his or her left or right eye gently and look directly at the nurse's opposite eye Close the opposite eye to superimpose the field of vision Move a finger equidistant between the nurse and the client outside the field of vision Ask the client to report when he or she is able to see the finger The first step while assessing the visual level of the client is to direct the client to stand or sit 60 cm away at eye level. Next, the nurse should ask the client to gently close or cover one eye and look at the nurse's eye directly opposite. Then, the nurse should also close his or her right eye to superimpose the field of vision. After this, the nurse should move a finger equidistant between the nurse and the client outside the field of vision. Finally, the nurse should ask the client to report when he or she is able to see the finger.

The nurse is assessing a young client who presents with recurrent gastrointestinal disorders. On further assessment, the nurse learns that the client is experiencing job-related pressure. What is the most important nursing intervention for this client? 1 Educate the client on managing stress. 2 Teach the client to maintain a balanced diet. 3 Instruct the client to have regular health checkups. 4 Ask the client to use sunscreen when working outdoors.

Educate the client on managing stress. The client is experiencing job-related pressure, so the nurse should educate the client about managing stress as it is a lifestyle risk factor. Stress threatens both mental health and physical well-being. Stress is associated with illnesses such as heart disease, cancer, and gastrointestinal disorders. The nurse teaches the client to maintain a balanced diet as a primary preventive care to promote health. The nurse should instruct the client to have regular health checkups as a primary preventive measure. The nurse should ask the client to use sunscreen when working outdoors to avoid excess sun exposure and prevent skin cancer.

While conducting research on a novel anticoagulant, clients are randomly assigned to a control group and treatment groups. The experimental drug is given to the treatment group and the standard anticoagulant drug is given to the control group. The researchers measured the outcomes in both groups and concluded that the novel anticoagulant drug has a better outcome when compared to the standard drug. Which quantitative method does this belongs to?

Experimental Research

A client who has been admitted to the hospital with chest pain complains of shortness of breath, weakness, and vomiting. The nurse suspects cardiac arrest. Which site is the most appropriate place to check the client's pulse rate?

Femoral A client with chest pain, shortness of breath, weakness, and vomiting may be experiencing cardiac arrest. In a client with cardiac arrest, the most appropriate place to check the pulse rate is the femoral site, because other pulses may not be palpable at this time.

Arrange the steps of the bimanual deep palpation technique in a sequence. 1. Relax the sensing hand 2. Place the sensing hand on the skin 3. Apply pressure on the sensing hand 4. Depress the area to be examined to 2 inches 5. Place the active hand on the sensing hand

I don't know the correct order. Explanation was During a deep palpation, the area under the examination is depressed to 2 inches using one or both hands. When both the hands are used for palpation, the sensing hand is relaxed and placed over the client's skin. Then the active hand is placed over the sensing hand and pressure is applied on the sensing hand.

The registered nurse is teaching a nursing student about ways to minimize heat radiation. Which statements made by the nursing student indicate effective learning? Select all that apply.

I will apply an ice pack to the client." "I will cover the client with dark clothes." "I will instruct the client to lie in the fetal position." Applying an ice pack will increase conductive heat loss, which results in minimizing heat radiation. Wearing dark clothes and lying in the fetal position will minimize heat radiation. Removing extra clothes will increase heat radiation. Wearing sparsely woven clothes will enhance heat radiation.

The nurse teaching a health awareness class identifies which situation as being the highest risk factor for the development of a deep vein thrombosis (DVT)?

INACTIVITY A DVT, or thrombus, may form as a result of venous stasis. It may lodge in a vein and can cause venous occlusion. Inactivity is a major cause of venous stasis leading to DVT. Pregnancy and tight clothing are also risk factors for DVT secondary to inactivity.

A client is admitted to the hospital for an elective surgical procedure. The client tells a nurse about the emotional stress of recently disclosing being a homosexual to family and friends. What is the nurse's first consideration when planning care?

Identifying personal feelings toward this client

What is a basic concept associated with rehabilitation that the nurse should consider when formulating discharge plans for clients?

Immediate or potential rehabilitation needs are exhibited by clients with health problems.

Four days after abdominal surgery a client has not passed flatus and there are no bowel sounds. Paralytic ileus is suspected. What does the nurse conclude is the most likely cause of the ileus?

Impaired neural functioning

Allen's test procedure

In the modified Allen test, one hand is examined at a time:[2] The hand is elevated and the patient is asked to clench their fist for about 30 seconds. Pressure is applied over the ulnar and the radial arteries so as to occlude both of them. Still elevated, the hand is then opened. It should appear blanched (pallor may be observed at the finger nails). Ulnar pressure is released while radial pressure is maintained, and the color should return within 5 to 15 seconds. If color returns as described, Allen's test is considered to be normal. If color fails to return, the test is considered abnormal and it suggests that the ulnar artery supply to the hand is not sufficient.[2] This indicates that it may not be safe to cannulate or needle the radial artery.

Which age-related change should the nurse consider when formulating a plan of care for an older adult?

Increased sensitivity to glare & diminished sensation of pain

A nurse is caring for a client who underwent cardiac catheterization. The client's skin was found to be blanched, and there was formation of edema of 15.2 cm (1-6 inches) at the site of catheterization. Upon further assessment, the skin was found to be cool, and the client complains of tenderness. Which condition does the nurse expect?

Infiltration

Which error will result in false high diastolic readings while measuring a client's blood pressure during a physical examination? 1 Inflating the cuff too slowly 2 Wrapping the cuff too loosely 3 Applying the stethoscope too firmly 4 Repeating the assessment too quickly

Inflating the cuff too slowly Inflating or deflating the cuff too slowly will yield false high diastolic readings. Wrapping the cuff too loosely will result in false high systolic and diastolic values. Applying the stethoscope too firmly will result in false low diastolic readings. Repeating the assessment too quickly will result in false high systolic readings.

A registered nurse (RN) is performing a physical assessment of four clients with various medical conditions as shown in the chart. Which client is expected to have concavely curved nails?

Iron deficency aneimia

A nurse is supportive of a child receiving long-term rehabilitation in the home rather than in a health care facility. Why is living with the family so important to a child's emotional development?

It is where child's identity and roles are learned

A nurse is preparing to change a client's dressing. What is the reason for using surgical asepsis during this procedure?

Keeps the area free of microorganisms

The nurse is preparing to assess the four abdominal quadrants of a client who complains of stomach pain. When determining the order of the assessment, the nurse recognizes that it is important to assess the symptomatic quadrant when? 1 First 2 Second 3 Third 4 Last

Last The nurse should systematically assess the abdomen concluding with the symptomatic area. Pain may be elicited in the symptomatic area if assessed first, second, or third, causing the muscles in other abdominal areas to tighten. This would interfere with the assessment.

A client suffers hypoxia and a resultant increase in deoxygenated hemoglobin in the blood. What are the best sites to assess this condition? Select all that apply. 1 Lips 2 Sclera 3 Mouth 4 Sacrum 5 Nail beds 6 Shoulders

Lips Mouth Nail beds Prolonged hypoxia resulting in increased amounts of deoxygenated blood causes cyanosis, which can be best evaluated in lips, mouth, nail beds, and skin (in extreme conditions). Sclera is the site of assessment for jaundice, while shoulders are assessed to confirm the condition of erythema.

A pregnant woman in her second trimester arrived at the hospital for a general health checkup. The physician recommended a pelvic examination to the client. Which position is most suitable for assessing the client in this condition? 1 Sims position 2 Supine position 3 Lithotomy position 4 Dorsal recumbent position

Lithotomy position Lithotomy position provides maximum exposure to the female genitalia and easy examination of the region. Therefore this position is recommended for examining pregnant women. Sims position is indicated for rectal and vaginal examinations. Supine position is recommended for examining anterior thorax, lungs, breasts, axilla, heart abdomen, extremities, and pulse. Dorsal recumbent position is mainly indicated to examine the abdomen because it promotes abdominal relaxation.

A nurse is assessing an older adult during a regular checkup. Which findings during the assessment are normal?

Loss of turgor Decreased night vision Decreased mobility of ribs

The nurse is caring for an African American client with renal failure. The client states that the illness is a punishment for sins. Which cultural health belief does the client communicate?

Magicoreligious belief

When reviewing a drug to be administered, the nurse identifies that the package insert indicates that the Z-track injection technique should be used. Under what circumstance does the nurse expect that this technique will be necessary?

Medication is irritating to subcutaneous tissue and skin.

prone position

Muscuskeltosystem The position is only for assessing extension of the hip joint, skin, buttocks. Patients w/ respiratory diffulties do not tolerate this position well

A nurse assesses the vital signs of a 50-year-old female client and documents the results. Which of the following are considered within normal range for this client?

Oral temperature 98.2° F, Apical pulse 88 beats per minute and regular, Blood pressure 116/78 mm Hg while in a sitting position

a systemic skeletal condition in which there is a decreased bone mass and deterioration of bone tissue.

Osteoporosis is

The nurse discovers several palpable elevated masses on a client's arms. Which term most accurately describes the assessment findings?

Papules

When monitoring a client 24 to 48 hours after surgery, the nurse should assess for which problem associated with anesthetic agents?

Paralytic ileus

A nurse is reviewing a plan of care for a client who was admitted with dehydration as a result of prolonged watery diarrhea. Which prescription should the nurse question?

Parenteral albumin (Albuminar)

A client with internal bleeding is in the intensive care unit (ICU) for observation. At the change of shift an alarm sounds, indicating a decrease in blood pressure. What is the initial nursing action?

Perform an assessment of the client before resuming the change-of-shift report.

What is the most important factor relative to a therapeutic nurse-client relationship when a nurse is caring for a client who is terminally ill?

Personal feelings about terminal illness

What does a nurse consider the most significant influence on many clients' perception of pain when interpreting findings from a pain assessment?

Previous experience and cultural values Interpretation of pain sensations is highly individual and is based on past experiences, which include cultural values.

A nurse must establish and maintain an airway in a client who has experienced a near-drowning in the ocean. For which potential danger should the nurse assess the client?

Pulmonary edema Additional fluid from surrounding tissues will be drawn into the lung because of the high osmotic pressure exerted by the salt content of the aspirated ocean water; this results in pulmonary edema. Hypoxia and acidosis may occur after a near-drowning, not alkalosis. Renal failure is not a sequela of near-drowning. Hypovolemia occurs because fluid is drawn into the lungs by the hypertonic saltwater.

A client, who is in a late stage of pancreatic cancer, intellectually understands the terminal nature of the illness. Behaviors that indicate the client is emotionally accepting of impending death are that the client is

Revising the client's will and planning a visit to a friend

A client suspected to have a prostate disorder is encouraged to have a rectal examination. What position of the client will facilitate a rectal examination by the registered nurse (RN)?

Sims position In Sims position, hips and knees are flexed, which results in exposure of the rectal area. Therefore *Sims position is most suitable for performing rectal examinations*. A prone position helps in assessing extension of hips, skin, and buttocks. The dorsal recumbent position is predominantly indicated for abdominal assessment because it promotes abdominal muscle relaxation. The lateral recumbent position is indicated for detecting heart murmurs.

When nurses are conducting health assessment interviews with older clients, what step should be included?

Spend time in several short sessions to elicit more complete information from the client

a congenital condition in which both eyes do not focus on an object simultaneously. In this condition, the eyes appear crossed.

Strabismus is

A client's serum potassium level has increased to 5.8 mEq/L. What action should the nurse implement first?

Take vital signs and notify the charge nurse or health care provider

The nurse administers a pneumococcal vaccine to a 70-year-old client. The client asks "Will I have to get this every year like I do with the flu shot?" How should the nurse respond?

The Centers for Disease Control and Prevention recommend that adults be immunized with pneumococcal vaccine at age 65 years or older with a *single dose* of the vaccine; if the pneumococcal vaccine was received before 65 years of age or if there is the highest risk of fatal pneumococcal infection, revaccination should occur 5 years after the initial vaccination

The nurse is caring for a client who has lost an arm in a motor vehicle accident. Which reaction made by the client leads the nurse to realize that the client is in the withdrawal phase of adjusting to the change in body image?

The client recognizes the reality and becomes anxious.

The nurse is preparing discharge instructions for a client that acquired a nosocomial infection, Clostridium difficile. What should the nurse include in the instructions?

The infection causes diarrhea accompanied by flatus and abdominal discomfort

which part of the hand and fingers is used because it is more sensitive to vibrations

The palmar surface

Where is the location of the popliteal pulse

The palpation of the popliteal pulse is done on the popliteal artery, which is present in the posterior surface of the knee, as depicted in the first figure. The palpation of the femoral pulse is in the femoral artery, which is present in the right groin as seen in the second figure. The third figure depicts the palpation of the brachial pulse. The fourth figure shows the palpation of the ulnar pulse.

A client who does not understand English requires an interpreter. Which nursing student action may exacerbate health disparities?

The student talks only to the interpreter about the client A nurse should follow certain strategies while working with an interpreter for a client who does not understand English. The nurse should talk to the client about the client's condition and care and not to the interpreter. The interpreter may act as a client advocate and represent the client's needs to the nurse. The nurse should use a trained medical interpreter who has a health care background. The nurse should maintain eye contact with the client and obtain feedback to be certain that the client understands.

The nurse recognizes that what is the reason the faucets on the sinks in a client's room are considered contaminated?

They are touched by dirty hands when turning the water on.

dorsal recumbent position

Used to assess head & neck, anterior thorax, and lungs, breast, axillae, heart, abdomen Rationale: Position is used for ab assessment b/c it promotes relaxation of abdominal muscles Limitation: Pt with painful disorders are more comfortable with knees flexed

Which positioning of the hand would be appropriate for assessing turgor in a client?

Using fingers to pinch at sternum. While measuring position, consistency, and turgor, the client's body parts should be grasped lightly with the fingertips as depicted in figure 4. The palmar surface of the hand and finger pads as shown in figure 1 is more sensitive than the fingertips. These should be used to determine position, texture, size, consistency, masses, fluid, and crepitus. While assessing body temperature, the dorsal surface (or back) of the hand should be used as shown in figure 2. In figure 3, the palmar surface of the hand and fingers is used because it is more sensitive to vibrations.

While performing a physical assessment of a female client, the nurse positions the client in Sims' position. Which body system will be assessed in this position?

Vagina Rectum

An infant has usually tripled his or her birth weight by

around 1 year.

Maintain strict asepsis during open catheter irrigation to minimize

contamination and subsequent development of a urinary tract infection (UTI).

Sudden reinflation of groups of alveoli may produce

crackling sounds

A health care provider prescribes an antibiotic intravenous piggyback (IVPB) twice a day for a client with an infection. The health care provider prescribes peak and trough levels 48 and 72 hours after initiation of the therapy. The client asks the nurse why there is a need for so many blood tests. The nurse's best response is, "These tests will:

determine adequate dosage levels of the drug

While assessing body temperature, the ____ ______of the hand should be used.

dorsal surface (or back)

Wrapping the cuff too loosely will result in

false high systolic and diastolic values.

Repeating the assessment too quickly will result in

false high systolic readings.

Applying the stethoscope too firmly will result in

false low diastolic readings.

Spending time in several short sessions reduces client -

fatigue and compensates for a shortened attention span, which is common in the older adult.

The nurse assesses a client's pulse and documents the strength of the pulse as 3+. The nurse understands that this indicates the pulse is:

full

The client with blanched skin, edema of 15.2 cm, cool temperature, and pain at the site of catheterization has symptoms of

grade 2 infiltration.

SKIN COLOR VARIATIONS: Tan-Brown

increased amt of melanin causes: suntan, preg Assessment location: areas exposed to sun, face, nipples

lithotomy position

lying on back with legs raised and feet in stirrups Areas Assessed: Female genital and GU Rationale: position provides max exposure of female genitalia and facilities insertion of vag speculum Limitations: Position is embarrassing and uncomfortable, thus examiner min time pt spends in it. Keep pt well draped

supine position

lying on back, facing upward

Turgor indicates the elasticity of the skin. The ideal site to assess the skin for turgor in an older adult is back of the forearm or the sternal area. The back of the neck contains redundant skin and may not be reliable. The skin on the back of the hand is normally loose and thin; turgor assessed at that site may not be reliable. The palm of the hand is not an ideal site for the assessment of turgor.

true

Research suggests that children who are spanked tend to use aggressive behavior; as they grow older they learn their own behavior through their parents' behavior.

true

While demonstrating the method of measuring blood pressure to a student nurse, the registered nurse measures the blood pressure in a client as 130/80 mm Hg. After the demonstration, when the student nurse is measuring the blood pressure in the same client, it is found to be 120/90 mm Hg. What could be the possible reasons for this difference? Select all that apply.

-Poor fitting of the cuff -Deflating the cuff too quickly

A client develops an allergic reaction when a student nurse is performing a physical assessment. Which statement made by the student nurse in response to this incident indicates the need for further teaching?

"The client's first exposure to latex will cause a type IV allergic reaction."

A nurse who is working on a medical-surgical unit receives a phone call requesting information about a client who has undergone surgery. The nurse observes that the client requested a do not publish ("DNP") order on any information regarding condition or presence in the hospital. What is the best response by the nurse?

"We have no record of that client on our unit. Thank you for calling."

Lovett scale

0 T P F G N

A client presents with a shiny appearance of abdominal skin. The skin also has a taut appearance. Which condition may the client have? 1 Ascites 2 Cyanosis 3 Accidental injury 4 Bleeding disorder

1 Ascites Symptoms of ascites include a shiny and taut appearance of the abdominal skin. Cyanosis occurs when there is a bluish discoloration of the skin. Accidental injury and different types of bleeding disorders are characterized by bruises or needle marks on the skin.

While assessing a client with dehydration, the nurse notices diminished skin elasticity. Which portion of the hand is used to perform this assessment? 1 Fingertips 2 Pads of fingertips 3 Ulnar surface of hand 4 Palmer surface of finger pads

1 Fingertips The fingertips are used to palpate the skin for elasticity. The pads of the fingertips are used to palpate pulse amplitude. The ulnar surface of the hand is used to detect fremitus. The palmer surface of the fingertips is used to examine the thorax.

Which error will result in false high diastolic readings while measuring a client's blood pressure during a physical examination? 1 Inflating the cuff too slowly 2 Wrapping the cuff too loosely 3 Applying the stethoscope too firmly 4 Repeating the assessment too quickly

1 Inflating the cuff too slowly Inflating or deflating the cuff too slowly will yield false high diastolic readings. Wrapping the cuff too loosely will result in false high systolic and diastolic values. Applying the stethoscope too firmly will result in false low diastolic readings. Repeating the assessment too quickly will result in false high systolic readings.

A client being treated for influenza A (H1N1) is scheduled for a computed tomography (CT) scan. To ensure client and visitor safety during transport, the nurse should take which precaution? 1 Place a surgical mask on the client. 2 Other than standards precautions, no additional precautions are needed. 3 Minimize close physical contact. 4 Cover the client's legs with a blanket.

1 Place a surgical mask on the client. Nurses should provide influenza clients with facemasks to wear for source control and tissues to contain secretions when outside of their room. Special precautions such as facemasks should be taken to decrease the risk of further outbreak. Minimizing close physical contact is not indicated. Covering the client with a blanket is for comfort and privacy, not because of a transmission precaution.

The nurse is caring for an elderly client with dementia. Which client need should the nurse prioritize while providing care? 1 Safety 2 Self-esteem 3 Self-actualization 4 Love and belonging

1 Safety An elderly client with dementia has impaired cognition. Therefore the nurse should make arrangements such as applying bed side rails to ensure that the client's safety needs are met first. At this stage, self-esteem or factors that enhance confidence and self-worth are not as important as safety. Self-actualization is the ability to solve problems and being able to cope realistically, which is beyond the capacity of the client with dementia. All clients need to feel love and belonging; however, safety is the first priority for this client.

A nurse is assessing an older adult client. Which clinical findings are expected responses to the aging process? Select all that apply. 1 Slowed neurologic responses 2 Lowered intelligence quotient 3 Long-term memory impairment 4 Forgetfulness about recent events 5 Reduced ability to maintain an erection

1 Slowed neurologic responses 4 Forgetfulness about recent events 5 Reduced ability to maintain an erection Slowing of neurologic responses is part of the aging process. Memory for short-term situations and events is reduced. The ability of the male to attain and sustain an erection is reduced. There should not be a loss of intellectual ability. Memory of long-term experiences and events should not be impaired.

A nurse is assessing an older adult client. Which clinical findings are expected responses to the aging process? Select all that apply. 1 Slowed neurological responses 2 Lowered intelligence quotient 3 Long-term memory impairment 4 Forgetfulness about recent events 5 Reduced ability to maintain an erection

1 Slowed neurological responses 4 Forgetfulness about recent events 5 Reduced ability to maintain an erection Slowing of neurological responses is part of the aging process. Memory for short-term situations and events is reduced. The ability of the male to attain and sustain an erection is reduced. There should not be a loss of intellectual ability. Memory of long-term experiences and events should not be impaired.

While assessing a client's skin, a nurse notices that the skin is dry. What is the probable etiology of the condition? Select all that apply. 1 Use of hard soap 2 Frequent bathing 3 Use of tanning pills 4 Presence of an allergy 5 Use of petroleum products

1 Use of hard soap 2 Frequent bathing The use of hard soap and frequent bathing may result in dry skin. A skin allergy may result in skin rashes, but not dry skin. Using tanning pills and petroleum products may result in skin cancer.

Evidence-based nursing uses a variety of sources to support nursing practice. Which are sources of evidence-based practice? Select all that apply. 1. Theory 2 Research 3 Time studies 4 Clinical expertise 5 Accepted nursing rituals

1. Theory 2 Research 4 Clinical expertise Evidenced-based nursing care uses information gleaned from theory, research, expert opinion, client history and physical examination, client preferences and values, and the clinical expertise of the nurse. Time/motion studies are not used as a basis of evidenced-based practice. Accepted nursing rituals are not used as a basis of evidenced-based practice.

Which infant does the nurse anticipate to be of abnormal weight? 1. Age 4 months weight at birth 2.9 kg, current weight 6.1kg 2. Age 5 months, weight at birth 3.3 kg, current weight 8.5kg 3. Age 12 months, weight at birth 3.35kg, current weight 10kg 4. Age 11 months, weight at birth 3.4 kg current weight 10.3 kg

2 The average birth weight of a newborn is 3.2 to 3.4 kg. An infant usually doubles his or her birth weight at 4 to 5 months of age. Therefore, infant 2's weight of 8.5 kg at 5 months is abnormal. Infant 1, weighing 6.1 kg, is of a normal weight. An infant has usually tripled his or her birth weight by around 1 year. Therefore, infants 3 and 4 are experiencing normal weight gain.

A nurse teaches a client about various measures to protect against food-borne illness. Which statement by the client indicates a need for further teaching? 1 "I'll clean the inside of my refrigerator and microwave regularly." 2 "I'll wash my cooking utensils and cutting boards with tap water." 3 "I'll wash my hands with warm, soapy water before touching or eating food." 4 "I won't eat any leftovers in my refrigerator after they've been there for 5 days."

2 "I'll wash my cooking utensils and cutting boards with tap water." Eating leftovers that have been kept in a refrigerator for more than 2 days may result in a food-borne illness caused by microbial growth in the food. Cleaning the inside of the refrigerator and microwave regularly will help prevent microbial growth. Cooking utensils and cutting boards should be washed with hot, soapy tap water as a means of preventing food-borne illness. Washing the hands with warm, soapy water before touching or eating food is one technique for preventing food borne illness.

A client who is suspected of having tetanus asks a nurse about immunizations against tetanus. Before responding, what should the nurse consider about the benefits of tetanus antitoxin? 1 It stimulates plasma cells directly. 2 A high titer of antibodies is generated. 3 It provides immediate active immunity. 4 A long-lasting passive immunity is produced

2 A high titer of antibodies is generated. Tetanus antitoxin provides antibodies, which confer immediate passive immunity. Antitoxin does not stimulate production of antibodies. It provides passive, not active, immunity. Passive immunity, by definition, is not long-lasting.

A preschool-aged child is admitted to the pediatric unit for urinary diversion surgery. The ureters are transplanted to a section of the colon, with one end attached to the abdominal wall as an ileostomy. The parents ask the nurse for the name of the procedure. What is the nurse's response? 1 Cystostomy 2 Ileal conduit 3 Ureterosigmoidostomy 4 Cutaneous ureterostomy

2 Ileal conduit An ileal conduit is the transplantation of the ureters to a section of the colon with one end attached to the abdominal wall. A cystostomy is the opening into the bladder through the abdominal wall that allows urine to flow out. An ureterosigmoidostomy involves transplanting the ureter into the colon so the urine is excreted through the rectum. A cutaneous ureterostomy involves the surgical creation of an opening from the ureter to the skin surface of the abdomen.

After an eye assessment, the nurse finds that both of the client's eyes are not focusing on an object simultaneously and appear crossed. What could be the cause for this condition? 1 Loss of elasticity of the lens 2 Impairment of the extraocular muscles 3 Obstruction of the aqueous humor outflow 4 Progressive degeneration of the center of the retina

2 Impairment of the extraocular muscles Strabismus is a condition where the eyes appear crossed; this condition is caused by the impairment of the extraocular muscles. A loss of lens elasticity may lead to presbyopia, which causes impaired near vision. An obstruction of the aqueous humor outflow may lead to glaucoma. The progressive degeneration of the center of the retina indicates macular degeneration and leads to blurred central vision.

While assessing the eyes of a client, a healthcare provider notices there is an obstruction to the outflow of aqueous humor. Which additional finding might be noted to support a diagnosis of glaucoma? 1 Blurred central vision 2 Increased opacity of the lens 3 Elevated intraocular pressure 4 Changes in retinal blood vessels

3 Elevated intraocular pressure In glaucoma, there is an obstruction of the outflow of aqueous humor due to an intraocular structural damage, which may result from elevated intraocular pressure. Blurred central vision is seen in macular degeneration. Increased opacity of the lens may be seen in cataracts. Retinopathy may result from the changes in retinal blood vessels.

While awaiting the biopsy report before removal of a tumor, the client reports being afraid of a diagnosis of cancer. How should the nurse respond? 1 "Worrying is not going to help the situation." 2 "Let's wait until we hear what the biopsy report says." 3 "It is very upsetting to have to wait for a biopsy report." 4 "Operations are not performed unless there are no other options."

3 "It is very upsetting to have to wait for a biopsy report." "It is very upsetting to have to wait for a biopsy report" addresses the fact that the client's feelings of anxiety are valid. Stating that "Worrying is not going to help the situation" and "Let's wait until we hear what the biopsy report says" do not address the client's concerns and may inhibit the expression of feelings. Telling the client that operations are not performed unless there are no other options is irrelevant and does not address the client's concerns.

Which client body temperatures are indicative of moderate hypothermia? Select all that apply. 1 80° F (26.7° C) 2 84° F (28.9° C) 3 88° F (31.1° C) 4 92° F (33.3° C) 5 96° F (35.6° C)

3 88° F (31.1° C) 4 92° F (33.3° C) Moderate hypothermia is a body temperature between 86°F and 93.2°F (30° C to 34° C). Therefore clients with body temperatures between 88°F and 92°F (31.1° C to 33.3° C) have moderate hypothermia. Mild hypothermia is a body temperature between 93.2°F and 96.8°F (34° C to 36° C). Therefore clients with body temperatures of 96°F (35.6° C) have mild hypothermia. Body temperature below 86°F (30° C) indicates severe hypothermia.

The family of an older adult who is aphasic reports to the nurse manager that the primary nurse failed to obtain a signed consent before inserting an indwelling catheter to measure hourly output. What should the nurse manager consider before responding? 1 Procedures for a client's benefit do not require a signed consent. 2 Clients who are aphasic are incapable of signing an informed consent. 3 A separate signed informed consent for routine treatments is unnecessary. 4 A specific intervention without a client's signed consent is an invasion of rights

3 A separate signed informed consent for routine treatments is unnecessary. This is considered a routine procedure to meet basic physiologic needs and is covered by a consent signed at the time of admission. The need for consent is not negated because the procedure is beneficial. This treatment does not require special consent.

The nurse teaches a client that heart failure can best be described as: 1 A cardiac condition caused by inadequate circulating blood volume 2 An acute state in which the pulmonary circulation pressure decreases 3 An inability of the heart to pump blood in proportion to metabolic needs 4 A chronic state in which the systolic blood pressure drops below 90 mm Hg

3 An inability of the heart to pump blood in proportion to metabolic needs As the heart fails, cardiac output decreases; eventually the decrease will reach a level that prevents tissues from receiving adequate oxygen and nutrients. Heart failure is related to an increased, not decreased, circulating blood volume. The condition may be acute or chronic; the pulmonary pressure increases and capillary fluid is forced into the alveoli. The blood pressure usually does not drop; the condition may be acute or chronic.

A nurse is teaching a parenting class. What should the nurse suggest about managing the behavior of a young school-age child? 1 Avoid answering questions. 2 Give the child a list of expectations. 3 Be consistent about established rules. 4 Allow the child to plan the day's activities.

3 Be consistent about established rules. Because of a short attention span and distractibility, consistent limit setting is essential toward providing an environment that promotes concentration, prevents confusion, and minimizes conflicts. Questions should be answered, but the answers should not be judgmental. A list of expectations may be overwhelming at this age. Parents need to assist children with routine tasks; children this age may not be concerned with time frames. STUDY TIP: Focus your study time on the common health problems that nurses most frequently encounter.

The nurse assesses an elderly client with a diagnosis of dehydration and recognizes which finding as an early sign of dehydration? 1 Sunken eyes 2 Dry, flaky skin 3 Change in mental status 4 Decreased bowel sounds

3 Change in mental status Older adults are sensitive to changes in fluid and electrolyte levels, especially sodium, potassium, and chloride. These changes will manifest as a change in mental status and confusion. It is difficult to assess dehydration in older adults based on sunken eyes, dry skin, and decreased bowel sounds because these can be prominent as general normal findings in the elderly client. Test-Taking Tip: The computerized NCLEX exam is an individualized testing experience in which the computer chooses your next question based on the ability and competency you have demonstrated on previous questions. The minimum number of questions will be 75 and the maximum 265. You must answer each question before the computer will present the next question, and you cannot go back to any previously answered questions. Remember that you do not have to answer all of the questions correctly to pass.

What should the nurse do when the defining characteristics of assessment data for a client can apply to more than one diagnosis? Select all that apply. 1 Reassess the client. 2 Reject all diagnoses. 3 Gather more information. 4 Identify related factors. 5 Review all defining characteristics.

3 Gather more information. 4 Identify related factors. 5 Review all defining characteristics.

The nurse recognizes that a common conflict experienced by older adults is the conflict between what? 1 Youth and old age 2 Retirement and work 3 Independence and dependence 4 Wishing to die and wishing to live

3 Independence and dependence A common conflict confronting older adults is between the desire to be taken care of by others and the desire to be in charge of their own destiny. The conflict between the young and old age may occur but is not common. The conflict between the retirement and working may occur but is not common. The conflict between wishing to die and wishing to live may occur but is not common.

The nurse is caring for a client that is on a low carbohydrate diet. With this diet, there is decreased glucose available for energy, and fat is metabolized for energy resulting in an increased production of which substance in the urine? 1 Protein 2 Glucose 3 Ketones 4 Uric acid

3 Ketones As a result of fat metabolism, ketone bodies are formed and the kidneys attempt to decrease the excess by filtration and excretion. Excessive ketones in the blood can cause metabolic acidosis. A low carbohydrate diet does not cause increased protein, glucose, or uric acid in the urine.

A nurse receives abnormal results of diagnostic testing. What action should the nurse take first? 1 Inform the client of the results. 2 Ensure that the results are placed in the client's medical record. 3 Notify the client's health care provider of the results. 4 Obtain results of the other lab tests that were performed.

3 Notify the client's health care provider of the results. The nurse is most ethically and legally accountable for reporting diagnostic testing results to the client's health care provider, whether the results are normal or, more important, abnormal. Informing the client of the results is an incorrect action in this situation. Placing the results in the client's record and obtaining normal values of the results from the lab are acceptable actions for the nurse after notifying the health care provider of the abnormal results.

A client is admitted to the hospital with severe diarrhea, abdominal cramps, and vomiting after eating. These symptoms have lasted 5 days. Upon further assessment, the primary healthcare provider finds that the symptoms occurred after the client ate eggs, salad dressings, and sandwich fillings. Which food borne disease would be suspected in this client? 1 Listeriosis 2 Shigellosis 3 Salmonellosis 4 Staphylococcus

3 Salmonellosis A client with salmonellosis will experience severe diarrhea, abdominal cramps, and vomiting; these symptoms last as long as 5 days after the intake of contaminated food. This disorder may be caused by Salmonella typhi or Salmonella paratyphi. The causative organism is usually present in such foods as eggs, salad dressings, and sandwich fillings. A client with listeriosis will experience severe diarrhea, fever, headache, pneumonia, meningitis, and endocarditis 3 to 21 days after infection. The symptoms of shigellosis range from cramps and diarrhea to a fatal dysentery that lasts for 3 to 14 days. Pain, vomiting, diarrhea, perspiration, headache, fever, and prostration lasting for 1 or 2 days are the symptoms of a Staphylococcus infection. Test-Taking Tip: Identify option components as correct or incorrect. This may help you identify an incorrect answer.

Which assessment should the nurse exclude when dealing with a client with receptive and expressive aphasia? 1 Ask the client to read simple sentences aloud 2 Point to a familiar object and ask the client to name it 3 Test the mental status by asking for feedback from the client 4 Ask the client to respond to simple verbal commands such as "Stand up"

3 Test the mental status by asking for feedback from the client Receptive and expressive aphasia are the two types of aphasia. A client with receptive is unable to understand written or verbal speech. A client with expressive aphasia understands written and verbal speech but cannot write or speak appropriately. A client with aphasia may not have the mental ability to give feedback; asking for feedback is ineffective. Asking the client to read simple sentences aloud is an effective way of dealing with this client. Pointing to a familiar object and asking the client to name it is also effective. A client with aphasia can understand simple verbal commands.

While talking with a 4-year-old child a nurse notes that the child is shy and stutters. What should the nurse recall about stuttering in a 4-year-old child? 1 It results from an emotional problem. 2 It is a sign of delayed neural development. 3 This is characteristic behavior in preschoolers. 4 This is an indication of permanent speech impairment.

3 This is characteristic behavior in preschoolers. Stuttering occurs because the child's advancing mental ability and level of comprehension exceed vocabulary acquisitions during the preschool years. Stuttering is not a sign of an emotional problem at this age, nor is it a sign of delayed neural development. Stuttering is common in the preschool years; when ignored, usually it disappears.

Following assessment, a nurse documents auscultation of course rhonchi in the anterior upper lung fields bilaterally that clears with coughing. What would be the cause of these sounds? 1 Parietal pleura rubbing against visceral pleura 2 Random, sudden reinflation of groups of alveoli 3 Turbulence due to muscular spasm and fluid or mucus in the larger airways 4 High-velocity airflow through severely narrowed or an obstructed airway

3 Turbulence due to muscular spasm and fluid or mucus in the larger airways Loud, low pitched, rumbling coarse sounds heard over the trachea and bronchi are due to turbulence caused by muscular spasm when fluid or mucous is present in the larger airways. Pleural rub produces a dry or grating quality sound, best heard in the lower portion of the anterior lateral lung. Random and sudden reinflation of groups of alveoli produces crackling sounds predominantly heard in the left and right lung bases. High-velocity airflow through severely narrowed or obstructed airways results in a wheezing sound heard all over the lung.

A client undergoing presurgical testing before a total abdominal hysterectomy says to the nurse, "After I have this surgery I know my husband will never come near me again." The nurse's best initial response is: 1 "You're underestimating his love for you." 2 "You're wondering about the effect on your sexual relations." 3 "You're worried that the surgery will change how others see you." 4 "You're concerned about how your husband will respond to your surgery."

4 "You're concerned about how your husband will respond to your surgery." Stating that the client is concerned about how her husband will respond to her surgery is an open-ended response that encourages further discussion without focusing on an area that the nurse, not the client, feels is the problem. Accusing the client of underestimating her husband's love denies the client's feeling and may cause feelings of guilt for questioning the partner's love. Wondering about the effect on sexual relations is too specific; the nurse does not have enough information to come to this conclusion. Worrying that the surgery will change how others see the client shifts the focus from the client's voiced concerns; the client specifically referred to her husband, not others.

A client has a history of a persistent cough, hemoptysis, unexplained weight loss, fatigue, night sweats, and fever. Which risk should be assessed? 1 Lung cancer 2 Cerebrovascular disease 3 Cardiopulmonary alterations 4 Human immunodeficiency virus (HIV) infection

4 Human immunodeficiency virus (HIV) infection A client with a history of persistent cough, hemoptysis, unexplained weight loss, fatigue, night sweats, or fever may have a human immunodeficiency virus (HIV) infection or tuberculosis. Lung cancer and cerebrovascular disease are risks to be assessed in the client with a history of tobacco or marijuana use. Cardiopulmonary alterations may be present in a client with a persistent cough (productive or nonproductive), sputum streaked with blood, or voice changes.

While performing a physical assessment of a client, a nurse notices patchy areas with loss of pigmentation on the skin, hands, and arms. What is the probable etiology for this condition? 1 Anemia 2 Pregnancy 3 Lung disease 4 Autoimmune disease

4 Autoimmune disease Patchy areas with loss of pigmentation on skin, hands, and arms are due to vitiligo, which is caused by an autoimmune or congenital disease. Anemia results in pallor due to a reduced amount of oxyhemoglobin. A tan-brown color of the skin is noticed in pregnancy due to an increased amount of melanin. Lung disease or heart failure can cause cyanosis due to an increased amount of deoxygenated hemoglobin. Test-Taking Tip: You have at least a 25% chance of selecting the correct response in multiple-choice items. If you are uncertain about a question, eliminate the choices that you believe are wrong and then call on your knowledge, skills, and abilities to choose from the remaining responses.

A nurse is observing the activities of 5-year-old children in the playroom. What typical play behavior does the nurse expect? 1 Parallel 2 Ritualistic 3 Aggressive 4 Cooperative

4 Cooperative Five-year-old children can interact with their peers and are beginning to learn how to share, be cooperative, and follow simple rules. Cooperative play is characteristic of 5-year-olds. Parallel play is characteristic of 2-year-olds. "Ritualistic" and "aggressive" are labels given to types of behavior, not types of play.

A pregnant woman in her second trimester arrived at the hospital for a general health checkup. The physician recommended a pelvic examination to the client. Which position is most suitable for assessing the client in this condition? 1 Sims position 2 Supine position 3 Lithotomy position 4 Dorsal recumbent position

4 Dorsal recumbent position Lithotomy position provides maximum exposure to the female genitalia and easy examination of the region. Therefore this position is recommended for examining pregnant women. Sims position is indicated for rectal and vaginal examinations. Supine position is recommended for examining anterior thorax, lungs, breasts, axilla, heart abdomen, extremities, and pulse. Dorsal recumbent position is mainly indicated to examine the abdomen because it promotes abdominal relaxation.

While assessing the client's skin, a nurse notices a skin condition, the pathophysiology of which involves increased visibility of oxyhemoglobin caused by an increased blood flow due to capillary dilation. Which condition is associated with this client? 1 Pallor 2 Vitiligo 3 Cyanosis 4 Erythema

4 Erythema Erythema occurs due to an increased visibility of oxyhemoglobin, which is caused by increased blood flow. Pallor is caused by a reduced amount of oxyhemoglobin or a reduced visibility of oxyhemoglobin. Vitiligo is a pigmentation disorder caused by autoimmune diseases. Cyanosis is a bluish discoloration of the skin around the lips; this occurs due to an increased amount of deoxygenated hemoglobin in the blood.

A client who has been admitted to the hospital with chest pain complains of shortness of breath, weakness, and vomiting. The nurse suspects cardiac arrest. Which site is the most appropriate place to check the client's pulse rate? 1 Ulnar 2 Radial 3 Brachial 4 Femoral

4 Femoral A client with chest pain, shortness of breath, weakness, and vomiting may be experiencing cardiac arrest. In a client with cardiac arrest, the most appropriate place to check the pulse rate is the femoral site, because other pulses may not be palpable at this time. The ulnar site is used to assess the status of circulation to the hand and also used to perform the Allen test. The radial site is commonly used to assess the character of the pulse peripherally and to assess the status of the circulation to the hand. The brachial site is used to assess the status of the circulation to the client's lower arm or the blood pressure is being auscultated. Test-Taking Tip: Assessing the condition of the client will help you determine which site is the appropriate place to check the client's pulse rate.

A nurse is discussing weight loss with an obese individual with Ménière disease. Which suggestion by the nurse is most important? 1 Limit intake to 900 calories per day. 2 Enroll in an exercise class. 3 Get involved in diversionary activities when there is an urge to eat. 4 Keep a diary of all foods eaten each day.

4 Keep a diary of all foods eaten each day. Keeping a record of what one eats helps to limit nonconscious and nervous eating by making the individual aware of intake. Limiting calories to 900 per day is a severe restriction that requires a primary healthcare provider's prescription. Exercise causes rapid head movements, which may precipitate a Ménière attack. Although diversionary activities are a therapeutic intervention, the nurse first should make suggestions that help increase the client's awareness of personal eating habits.

Which sites would the nurse prefer while assessing for turgor in an older adult? Select all that apply. 1 Back of the neck 2 Back of the hand 3 Palm of the hand 4 On the sternal area 5 Back of the fore arm

4 On the sternal area 5 Back of the fore arm Turgor indicates the elasticity of the skin. The ideal site to assess the skin for turgor in an older adult is back of the forearm or the sternal area. The back of the neck contains redundant skin and may not be reliable. The skin on the back of the hand is normally loose and thin; turgor assessed at that site may not be reliable. The palm of the hand is not an ideal site for the assessment of turgor. Test-Taking Tip: Be alert for details about what you are being asked to do. In this Question Type, you are asked to select all options that apply to a given situation or patient. All options likely relate to the situation, but only some of the options may relate directly to the situation.

Norm HR for todlers - Children 1 to 2 years old:

80 to 140 beats per minute

Norm Heart rate for infants (1 to 11 months old):

80 to 160 beats per minute

The nurse is preparing to assess the four abdominal quadrants of a client who complains of stomach pain. When determining the order of the assessment, the nurse recognizes that it is important to assess the symptomatic quadrant when? 1 First 2 Second 3 Third 4 Last

4. Last The nurse should systematically assess the abdomen concluding with the symptomatic area. Pain may be elicited in the symptomatic area if assessed first, second, or third, causing the muscles in other abdominal areas to tighten. This would interfere with the assessment.

A client has been diagnosed with type 1 Diabetes Mellitus. When providing instructions on sharps disposal, the nurse should instruct the client to place the syringes in:

A plastic liquid detergent bottle with a screw-top lid

Lateral Recumbant (recovery) position

Areas Assessed: Heart Position aids in detecting murmurs Pt with respiratory difficultes do not tolerate this postistion well

The student nurse prepares a concept map while caring for a client recovering from surgery. What is the first step that the student nurse should take when preparing the concept map?

Arrange cues into clusters that form patterns

The nurse is assessing a client who arrived at the healthcare facility for an appointment. Which action by the nurse will be beneficial during the interview?

Asking about the client's current concerns

Sitting position

sitting upright provides full expansion of lungs and better visualization of symmetry of upper body parts. Physically weakened pt is sometimes unable to sit. Use supine pos. w/ head of bed elevated

When the abuse is related to an intimate partner, the nurse may observe

strangulation marks on the neck from rope burns or bruises.

Which Korotkoff sound represents the diastolic pressure in children?

the fourth

Semi-Fowler's Position

the head of the bed is raised 30 degrees; or the head of the bed is raised 30 degrees and the knee portion is raised 15 degrees

After abdominal surgery a client reports pain. What action should the nurse take first?

Determine the characteristics of the pain

The nurse expects a client with an elevated temperature to exhibit what indicators of pyrexia?

Flushed face, increased pulse rate

Nearsightedness, a refractive error in which rays of light and focus *in front of the retina*. Persons are able to clearly see close objects but not distant objects

Myopia

Which sites would the nurse prefer while assessing for turgor in an older adult? Select all that apply.

On the sternal area Back of the forearm

A registered nurse (RN) must assess the body temperature of a client with a history of epilepsy. Which site for measuring temperature is contraindicated in this client?

Oral cavity

A client diagnosed with tuberculosis is taking isoniazid (INH). To prevent a food and drug interaction, the nurse should advise the client to avoid:

Red wine

A nurse is preparing to assess the heart of a client during a routine health checkup. Which positioning of the client would be appropriate to assess the murmurs of the heart effectively?

The client should lie in the lateral recumbent position so that the nurse can effectively detect heart murmurs (as shown in figure 2). The supine position provides easy access to the pulse sites (shown in figure 1). The client should be placed in the dorsal recumbent position (figure 3) for abdominal assessment. Sims' position (figure 4) is used so the nurse can assess the rectum and vagina.

What is the duty of a nurse while caring for a client?

The nurse should determine the client's care preferences.

Which position is used to assess the extension of the hip joint and buttocks?

To assess the extension of hip joint and buttocks, the client should be positioned in prone position (as seen in the second figure). The dorsal recumbent position (as seen in the first figure) is used for an abdominal assessment. The lateral recumbent position (as seen in the third figure) is used to assess murmurs. The supine position (as seen in the fourth figure) is used to assess the heart, abdomen, extremities, and pulses.

Avoid open catheter irrigation unless it is needed to relieve or prevent obstruction. The procedure requires breaking or opening the connection between the catheter and drainage bag of a closed urinary drainage system.

True, this leaves greater oppurtunity for infection

Edema results from

a direct trauma to the tissue or by impaired venous return.

Glasgow Coma Scale (GCS)

a scale used to assess the consciousness of a patient upon physical examination, typically in patients with neurological concerns or complaints 3 categories 1. Eyes Open Spontaneously - 4pts To speech - 3pts To pain - 2 pts None - 1 pt 2. Best Verbal Response Oriented - 5pts Confused - 4pts Inappropriate words - 3pts Incomprehensible sounds - 2 pts None - 1pt 3. Best Motor Response Obey's Commands 6 pts Localized Pain - 5pts Flexion Withdrawl - 4pts Abnormal Flexion - 3pts Abnormal Extension - 2 pts Flaccid 1 pt Total score - 3 to 15

During the _______ _______, the client and family go through a grieving period as they acknowledge the change in physical appearance.

acknowledgement phase, At the end of the acknowledgement phase, they learn to accept the loss.

The fifth Korotkoff sound represents the diastolic pressure in

adults and adolescents.

Heart and lung abnormalities such as chronic obstructive pulmonary disease cause

clubbing of the nail beds.

Conditions such as syphilis and iron deficiency anemia cause

concavely curved nails, called koilonychia

After losing a limb in a MVA Initially, the client is in a state of shock and

depersonalizes the change. The client talks as if another person is affected by the change.

The Allen's test is

is a medical sign used in physical examination of arterial blood flow to the hands

When assessing a client's blood pressure, the nurse notes that the blood pressure reading in the right arm is 10 mm Hg higher than the blood pressure reading in the left arm. The nurse understands that this finding:

is a normal occurrence

prone position

lying on abdomen, facing downward (head may be turned to one side)

Staying isolated and not communicating with others are behavioral findings that may be related to: Weight loss and sunken eyes may be a physical finding for:

older adult abuse for both

A barrell shaped chest is a characteristic feature in an

older adult who smokes and has chronic lung disease.

While measuring _______, _______, and ______the client's body parts should be grasped lightly with the fingertips

position, consistency, and turgor,

The client in the _______ _______ is ready to learn how to adapt to the change in body image through use of prosthesis or changing lifestyles and goals.

rehabilitation stage

A blowing or swishing sound occurs in the

second Korotkoff sound.

The nurse is performing a breast assessment. Which statement made by the client indicates the risk of breast cancer? Select all that apply.

- "My first child was born when I was 32." - "I noticed a slight discharge from a nipple." - "I consume two to four glasses of alcohol a day."

A nurse is caring for an older adult who is taking acetaminophen (Tylenol) for the relief of chronic pain. Which substance is most important for the nurse to determine if the client is taking because it intensifies the most serious adverse effect of acetaminophen? 1 Alcohol 2 Caffeine 3 Saw palmetto 4 St. John's wort

1 Alcohol Too much ingestion of alcohol can cause scarring and fibrosis of the liver. Eighty-five to 95% of acetaminophen is metabolized by the liver. Acetaminophen and alcohol are both hepatotoxic substances. Metabolites of acetaminophen along with alcohol can cause irreversible liver damage. Caffeine affects (stimulates) the cardiovascular system, not the liver. In addition, caffeine does not interact with acetaminophen. Saw palmetto is not associated with increased liver damage when taking acetaminophen. It often is taken for benign prostatic hypertrophy because of its antiinflammatory and antiproliferative properties in prostate tissue. St. John's wort is classified as an antidepressant and is not associated with increased liver damage when taking acetaminophen. However, it does decrease the effectiveness of acetaminophen.

A nurse is reviewing how a hyperglycemic client's blood glucose can be lowered. The nurse recalls that the chemical that buffers the client's excessive acetoacetic acid is: 1 Potassium 2 Sodium bicarbonate 3 Carbon dioxide 4 Sodium chloride

2 Sodium bicarbonate Sodium bicarbonate is a base and one of the major buffers in the body. Potassium, a cation, is not a buffer; only a base can buffer an acid. Carbon dioxide is carried in aqueous solution as carbonic acid (H2CO3); an acid does not buffer another acid. Sodium chloride is not a buffer; it is a salt.

A client with a borderline personality disorder becomes hostile and calls the nurse names. When the nurse denies privileges, the client states that the nurse is uncaring. How can the nurse be most therapeutic in this situation? 1 By helping the client identify feelings 2 By increasing the client's limits on privileges 3 By avoiding the client until the hostility is resolved 4 By advising the client how to approach people differently

1 By helping the client identify feelings To be most therapeutic the nurse needs to help the client identify his feelings, thereby aiding self-understanding. Increasing the limits on the client's privileges is a hostile response by the nurse. Avoiding the client will increase the client's hostility. The nurse should never give advice to clients; the nurse's role is to facilitate the client's problem-solving abilities.

A nurse is evaluating the effectiveness of treatment for a client with excessive fluid volume. What clinical finding indicates that treatment has been successful? 1 Clear breath sounds 2 Positive pedal pulses 3 Normal potassium level 4 Increased urine specific gravity

1 Clear breath sounds Excess fluid can move into the lungs, causing crackles; clear breath sounds support that treatment was effective. While it may make palpation more difficult, excess fluid will not diminish pedal pulses. A normal potassium level can be maintained independently of fluid excess correction. As the client excretes excess fluid, the urine specific gravity will decrease, not increase.

A health care provider prescribes an antibiotic intravenous piggyback (IVPB) twice a day for a client with an infection. The health care provider prescribes peak and trough levels 48 and 72 hours after initiation of the therapy. The client asks the nurse why there is a need for so many blood tests. The nurse's best response is, "These tests will: 1 Determine adequate dosage levels of the drug." 2 Detect if you are having an allergic reaction to the drug." 3 Permit blood culture specimens to be obtained when the drug is at its lowest level." 4 Allow comparison of your fever to when the blood level of the antibiotic is at its highest."

1 Determine adequate dosage levels of the drug." Drug dose and frequency are adjusted according to peak and trough levels to enhance efficacy by maintaining therapeutic levels. Peak and trough levels reveal nothing about allergic reactions. Blood cultures are obtained when the client spikes a temperature; they are not related to peak and trough levels of an antibiotic. A sustained decrease in fever is the desired outcome, not reduction just at peak serum levels of the medication.

A client is admitted with cellulitis of the left leg and a temperature of 103° F. The primary health care provider prescribes intravenous (IV) antibiotics. Before instituting this therapy, the nurse should: 1 Determine the client's allergies 2 Apply a warm, moist dressing over the cellulitis 3 Measure the amount of swelling in the client's left leg 4 Obtain the results of the culture and sensitivity tests

1 Determine the client's allergies Drug hypersensitivity and anaphylaxis are most common with antimicrobial agents. Applying a warm, moist dressing over the area is a dependent function; it is not crucial to starting antibiotic therapy. Measuring the amount of swelling in the client's leg is an important assessment, but it is not crucial to starting antibiotic therapy. Withholding treatment until culture results are available may extend the infection.

A client is diagnosed with thrombophlebitis. The client states, "I am worried about getting a clot in my lungs that will kill me." The nurse's initial response should be to: 1 Discuss the client's concerns 2 Clarify the misconception 3 Explain measures to prevent pulmonary emboli 4 Teach recognition of early symptoms of pulmonary emboli

1 Discuss the client's concerns Addressing the client's feelings and then exploring preventive measures should reduce anxiety. The risk of a pulmonary embolus is a real concern, not a misconception, associated with thrombophlebitis. Explaining measures to prevent a pulmonary embolus is not the client's concern; this response does not address the client's feelings concerning the risk of sudden death. Teaching recognition of early signs and symptoms of pulmonary emboli disregards the client's expressed fears and may increase anxiety.

A nurse is teaching Hands Only Basic Life Support for adults in the community. What should the rescuer do first after determining that the person is not responding and the emergency medical system has been activated? 1 Identify the absence of pulse. 2 Give two rescue breaths with a CPR mask. 3 Perform the head tilt-chin lift maneuver. 4 Perform chest compression at a rate of 100/min

1 Identify the absence of pulse. Once it is verified that the person is unresponsive and the emergency medical system has been activated, then whether the client is breathing should be established. Rescue breaths are not given with the hands-only basic life support method of CPR. Chest compressions are initiated as soon as it is identified that the person is not breathing; they are given at a rate of 100/min, to a depth of 2 inches each for 2 minutes, allowing full recoil between compressions. This quickly circulates the blood.

Alternative therapy measures have become increasingly accepted within the past decade, especially in the relief of pain. Which methods qualify as alternative therapies for pain? Select all that apply. 1 Prayer 2 Hypnosis 3 Medication 4 Aromatherapy 5 Guided imagery

1 Prayer 2 Hypnosis 4 Aromatherapy 5 Guided imagery Prayer is an alternative therapy that may relax the client and provide strength, solace, or acceptance. The relief of pain through hypnosis is based on suggestion; also, it focuses attention away from the pain. Some clients learn to hypnotize themselves. Aromatherapy can help relax and distract the individual and thus increase tolerance for pain, as well as relieve pain. Guided imagery can help relax and distract the individual and thus increase tolerance for pain, as well as relieve pain. Analgesics, both opioid and nonopioid, long have been part of the standard medical regimen for pain relief, so they are not considered an alternative therapy.

The parents of a newborn ask the nurse about several areas of deep-blue coloring on their baby's lower back and buttocks. The nurse's response is based on the information that: 1 These areas usually are normal and will fade within the first year. 2 Color changes represent transient mottling that occurs when the baby is cold. 3 These are characteristic of the harlequin color change that occurs when the newborn lies on the side. 4 Discolorations are probably bruises requiring observation of the infant for the development of jaundice

1 These areas usually are normal and will fade within the first year. Areas of deep-blue coloring on the skin, often seen on the lower back and buttocks, are called Mongolian spots. Mongolian spots are a variation within the norm and disappear in the first year. Mottling caused by cold covers the entire body. The harlequin color change is not purple or blue and involves an entire half of the body. In this newborn these are expected findings; if the baby were light skinned, the possibility of bruises should be investigated.

A nurse is providing preoperative teaching for a client regarding use of an incentive spirometer and should include what instructions? 1 "Inhale completely and exhale in short, rapid breaths." 2 "Inhale deeply through the spirometer, hold it as long as possible, and slowly exhale." 3 "Exhale completely; take a slow, deep breath; hold it as long as possible, and slowly exhale." 4 "Exhale halfway, then inhale a rapid, small breath; repeat several times.

2 "Inhale deeply through the spirometer, hold it as long as possible, and slowly exhale." The correct procedure to maximize use of an incentive spirometer is to exhale completely, then take a slow, deep breath through the spirometer, and hold it as long as possible. This procedure will maximize inspiratory function by expanding the lungs. The client should practice using the incentive spirometer before surgery. Answer options 1 and 4 are completely inaccurate procedures for using an incentive spirometer. Option 3 is partially correct but does not state to use the incentive spirometer. When teaching clients, it is important to provide exact step-by-step instructions, thus not leaving out any critical points.

A client who had abdominal surgery asks the nurse about when the client can return to work after discharge. The most appropriate response by the nurse is: 1 "Not for at least two weeks." 2 "What type of work did you have in mind?" 3 "You can return to work soon if you know what it means to take it easy." 4 "You cannot return to work soon because you must get plenty of rest when you get home."

2 "What type of work did you have in mind?" The nurse must identify the client's work activities before an appropriate response can be made. The client probably can do light work that will not injure the surgical site. The response "You can return to work if you know what it means to take it easy" is vague and demeaning and gives little direction to the client. The client probably can do light work as well as rest.

A client states, "I keep my insulin in the refrigerator because that is where my parents kept it." What reason should the nurse include when explaining that insulin should be stored at room temperature? 1 Its potency and effectiveness are maximized. 2 Absorption is enhanced and local irritation is decreased. 3 It is more convenient and drawing insulin into the syringe is facilitated. 4 Adherence of insulin to the syringe and resistance upon injection are decreased.

2 Absorption is enhanced and local irritation is decreased. Insulin can be stored at room temperature for up to 1 month but must be kept away from heat or sunlight. Inappropriate storage of insulin can decrease its stability and therefore decrease its therapeutic action. Insulin that is close to body temperature prevents vasoconstriction at the site and decreases irritation of tissues. Although it is more convenient, this is not a valid rationale; temperature of the solution does not increase the viscosity of insulin. Neither adherence of insulin to the syringe or decreased resistance upon injection occurs; the temperature of the solution does not increase the viscosity of insulin.

A female client explains to the nurse that she sleeps until noon every day and takes frequent naps during the rest of the day. What should the nurse do initially? 1 Encourage her to exercise during the day. 2 Arrange a referral for a thorough medical evaluation. 3 Explain that this behavior is an attempt to avoid facing daily responsibilities. 4 Identify that the client is describing clinical findings associated with narcolepsy.

2 Arrange a referral for a thorough medical evaluation. This behavior is a sign of hypersomnia and the client needs a medical assessment; it frequently is caused by central nervous system damage or certain kidney, liver, or metabolic disorders. Exercise is appropriate for a client experiencing insomnia, not hypersomnia. This behavior is a sign of hypersomnia and medical causes should be ruled out before attributing it to a psychogenic cause. Narcolepsy consists of recurrent sudden waves of overwhelming sleepiness that occur during the day, even during activities such as eating or conversing.

A nurse manager is evaluating the performance of the registered nurse (RN) who is supervising unlicensed assistive personnel (UAP). What action indicates to the nurse manager that the RN needs further instruction? 1 Requests that the UAP take vital signs on the clients assigned to their team 2 Asks the UAP to assess the client's response to a respiratory treatment 3 Instructs the UAP to communicate to a client that the meal trays will be delayed 4 Collaborates with the UAP to determine the best time to ambulate a client

2 Asks the UAP to assess the client's response to a respiratory treatment Assessing a client's response to care is beyond the scope of a nursing assistant's role. Evaluating a client's response to care is within the role of the registered nurse . Informing a patient about a meal tray delay and taking routine vital signs is part of UAP's scope of practice. The UAP is an essential part of the health care team, and all members of the team should collaborate to provide optimum patient care.

The way individuals cope with an unexpected hospitalization depends on many factors. However, the one that is most significant is: 1 Cognitive age 2 Basic personality 3 Financial resources 4 General physical health

2 Basic personality Lifelong coping styles are most important in how a person will deal with stress. Age may influence defense mechanisms but lifelong coping styles will most significantly affect a person's behavior. Financial resources are a factor to be considered, but past coping ability is the most significant factor to predict future coping. General physical health is a factor to be considered, but past coping ability is the most significant factor to predict future coping.

A client becomes anxious after being scheduled for a colostomy. What is the most effective way for the nurse to help the client? 1 Administer the prescribed as needed (prn) sedative. 2 Encourage the client to express feelings. 3 Explain the postprocedure course of treatment. 4 Reassure the client that there are others with this problem.

2 Encourage the client to express feelings. Communication is important in relieving anxiety and reducing stress. Administering the prescribed prn sedative does not acknowledge the client's feelings and does not address the source of the anxiety. Learning is limited when anxiety is too high. The focus should be on the client, not others. Reassurance may cut off communication and deny emotions.

A health care provider prescribes contact isolation for a client with an infected surgical incision. After being taught about isolation, the client is seen walking out of the room to the unit's desk. What is the most effective nursing intervention after the client is escorted back to the room? 1 Ensure regular visits by staff members. 2 Explore what isolation means to the client. 3 Reteach the isolation procedure to the client. 4 Report the situation to the infection control nurse.

2 Explore what isolation means to the client. The client must be taught what contact precautions entail. Communication facilitates joint solution of the problem; the nurse must assess the client's understanding and perceptions before solutions to the problem can be attempted. Ensuring regular visits by staff members will not collect data about why the client is leaving the room. Reteaching the isolation procedure to the client may be needed but not until further assessment has determined the reason why the client left the room. Reporting the situation to the infection control nurse abdicates the responsibility of the primary nurse.

The nurse expects a client with an elevated temperature to exhibit what indicators of pyrexia? Select all that apply. 1. Dyspnea 2 Flushed face 3 Precordial pain 4 Increased pulse rate 5 Increased blood pressure

2 Flushed face 4 Increased pulse rate Increased body heat dilates blood vessels, causing a flushed face. The pulse rate increases to meet increased tissue demands for oxygen in the febrile state. Fever may not cause difficult breathing. Pain is not related to fever. Blood pressure is not expected to increase with fever.

A pain scale of 1 to 10 is used by a nurse to assess a client's degree of pain. The client rates the pain as an 8 before receiving an analgesic and a 7 after being medicated. What conclusion should the nurse make regarding the client's response to pain medication? 1 Client has a low pain tolerance. 2 Medication is not adequately effective. 3 Medication has sufficiently decreased the pain level. 4 Client needs more education about the use of the pain scale.

2 Medication is not adequately effective. The expected effect should be more than a 1-point decrease in the pain level. Identifying whether a client has a low pain tolerance cannot be determined with the data available. The medication has not achieved an adequate response; pain generally is considered to be tolerable if it is 4 or below on a pain scale of 1 to 10. Determining that the client needs more education about the use of the pain scale cannot be determined with the data available.

A mother and her 5-year-old daughter have been referred to a child advocacy center for a forensic pediatric sexual examination. Before the child is examined or interviewed, the mother gives a detailed history, relaying her suspicion that the child's maternal grandfather sexually assaulted her. As the interview progresses, the mother suddenly says, "My father sexually molested me when I was a child, but I try not to think about it." What defense mechanism does the nurse recognize that the mother's statement demonstrates? 1 Introjection 2 Suppression 3 Passive aggression 4 Reaction formation

2 Suppression Suppression is voluntary refusal to admit an unacceptable idea or behavior. Introjection is the unconscious incorporation of wishes, values, and attitudes of others as if they were one's own. Passive-aggressive behavior is the expression of anger and hostility toward others in an indirect and nonassertive way. Reaction formation is the exact opposite of an unconscious feeling.

An adult child of a dying client says to the nurse in the nursing home, "I am so upset because my parent is always angry at me." The nurse's best initial response is "Your parent is: 1 frightened by impending death." 2 working through acceptance of the situation." 3 attempting to reduce your need for dependency." 4 hurt that you will not provide physical care at home."

2 working through acceptance of the situation." Understanding the stages leading to the acceptance of death may help the family member to understand the client's moods and anger. The parent may not be frightened unless stated by the client; some clients welcome death as a release from pain. It is unlikely that the parent is attempting to reduce the family member's need for dependency; anger is one of the stages of accepting death. It is an assumption by the nurse that the parent is hurt that the family member will not provide physical care at home unless stated by the client.

What is the most important factor relative to a therapeutic nurse-client relationship when a nurse is caring for a client who is terminally ill? 1. Knowledge of the grieving process 2. Personal feelings about terminal illness 3 Recognition of the family's ability to cope 4 Previous experience with terminally ill clients

2. Personal feelings about terminal illness To be effective in a relationship with a client, the nurse must know and understand personal feelings about terminal illness and death. Knowledge alone is not enough to ensure an effective nurse-client relationship. Although the family is an important part of a client's support system, the client's feelings are more important to the relationship. Previous experiences can be positive or negative and will not guarantee an effective nurse-client relationship.

A client whose wife recently died appears extremely depressed. The client says, "What's the use in talking? I'd rather be dead. I can't go on without my wife." What is the best response by the nurse? 1 "Would you rather be dead?" 2 "What does death mean to you?" 3 "Are you thinking about killing yourself?" 4 "Do you understand why you feel that way?"

3 "Are you thinking about killing yourself?" The response "Are you thinking about killing yourself?" is the most important assessment to make because suicide is a possibility with every depressed client. The client has already said that he would rather be dead, and the response addresses only part of the client's statement. The response "What does death mean to you?" is a philosophical approach that will not encourage discussion of feelings. The client is probably unable to explain why he feels the way he does.

Neomycin, 1 gram, is prescribed preoperatively for a client with cancer of the colon. The client asks why this is necessary. How should the nurse respond? 1 "It is used to prevent you from getting a bladder infection before surgery." 2 "It will decrease your kidney function and lessen urine production during surgery." 3 "It will kill the bacteria in your bowel and decrease the risk for infection after surgery." 4 "It is used to alter the body flora, which reduces spread of the tumor to adjacent organs."

3 "It will kill the bacteria in your bowel and decrease the risk for infection after surgery." Neomycin provides preoperative intestinal antisepsis. It is not administered to prevent bladder infection. Nephrotoxicity is an adverse, not a therapeutic, effect. Neomycin will not prevent metastasis of the tumor to other areas.

What information should the nurse give the client after removal of a foley catheter? 1 "Don't drink too much because you'll feel bloated." 2 "Measure your first void and then discard it." 3 "Void in the plastic measuring hat placed in the toilet and then contact the nurse." 4 "If you can't void in four hours, we'll have to reinsert the foley."

3 "Void in the plastic measuring hat placed in the toilet and then contact the nurse." The client's first void must be viewed and measured by the nurse to see if the color and integrity is normal and that the amount is sufficient. The client is encouraged to drink 2400 to 3000 mLs of fluid per day to stimulate the bladder. After the catheter is removed, the nurse needs to monitor for incontinence and urinary retention. The client also is expected to void at least 200 mL within six to eight hours after the foley has been removed. The client needs to be informed that some burning, frequency, and dribbling may occur following catheter removal.

A client has a right above-the-knee amputation after trauma sustained in a work-related accident. Upon awakening from surgery, the client states, "What happened to me? I don't remember a thing." What is the nurse's best response? 1 "Tell me what you think happened." 2 "You will remember more as you get better." 3 "You were in a work-related accident this morning." 4 "It was necessary to amputate your leg after the accident

3 "You were in a work-related accident this morning." The correct response is truthful and provides basic information that may prompt recollection of what occurred; it is a starting point. Asking the client to tell the nurse what happened ignores the client's question; avoidance may increase anxiety. Saying "you will remember more as you get better" ignores the client's question; the frustration of trying to remember will increase anxiety. Saying "it was necessary to amputate your leg after the accident" is too blunt for the initial response to the client's question; the client may not be ready to hear this at this time.

A nurse receives a subpoena in a court case involving a child. The nurse is preparing to appear in court. In addition to the state Nurse Practice Act and the American Nursing Association (ANA) Code for Nurses, what else should the nurse review? 1 Nursing's Social Policy Statement 2 State law regarding protection of minors 3 ANA Standards of Clinical Nursing Practice 4 References regarding a child's right to consent

3 ANA Standards of Clinical Nursing Practice The ANA Standards of Clinical Nursing Practice guidelines govern safe nursing practice; nurses are legally responsible to perform according to these guidelines. Nursing's Social Policy Statement explains what the public can expect from nurses, but it is not used to govern nursing practice. There are no data that indicate state law regarding protection of minors and references regarding a child's right to consent are necessary.

A client's chest tube has accidentally dislodged. What is the nursing action of highest priority? 1. Place the client in a left side-lying position. 2 Apply oxygen via non-rebreather mask. 3 Apply a petroleum gauze dressing over the site. 4 Prepare to reinsert a new chest tube.

3 Apply a petroleum gauze dressing over the site. A petroleum gauze dressing will prevent air from being sucked into the pleural space, causing a pneumothorax. The petroleum gauze dressing should be taped only on three sides to allow for excessive air to escape, preventing a tension pneumothorax. The physician should immediately be notified and the client assessed for signs of respiratory distress. Preparing to reinsert a new chest tube is not a priority of the nurse at this moment. Positioning the client on the left side will not make a difference in outcome. There is no indication that the client is experiencing respiratory distress.

A client that is scheduled for a surgical resection of the colon and creation of a colostomy for a bowel malignancy asks why preoperative antibiotics have been prescribed. The nurse explains that the primary purpose is to: 1 Decrease peristalsis. 2 Minimize electrolyte imbalance. 3 Decrease bacteria in the intestines. 4 Treat inflammation caused by the malignancy

3 Decrease bacteria in the intestines. To decrease the possibility of contamination, the bacteria count in the colon is lowered with antibiotics before surgery. Preoperative antibiotics do not have an effect on peristalsis, electrolyte balance, or treating inflammation.

A client is admitted with metabolic acidosis. The nurse considers that two body systems interact with the bicarbonate buffer system to preserve healthy body fluid pH. What two body systems should the nurse assess for compensatory changes? 1 Skeletal and nervous 2 Circulatory and urinary 3 Respiratory and urinary 4 Muscular and endocrine

3 Respiratory and urinary Increased respirations blow off carbon dioxide (CO2), which decreases the hydrogen ion concentration and the pH increases (less acidity). Decreased respirations result in CO2 buildup, which increases hydrogen ion concentration and the pH falls (more acidity). The kidneys either conserve or excrete bicarbonate and hydrogen ions, which helps to adjust the body's pH. The buffering capacity of the renal system is greater than that of the pulmonary system, but the pulmonary system is quicker to respond. Skeletal and nervous systems do not maintain the pH, nor do muscular and endocrine systems. Although the circulatory system carries fluids and electrolytes to the kidneys, it does not interact with the urinary system to regulate plasma pH.

A nurse teaches a postpartum client how to care for her episiotomy at home. What statement indicates to the nurse that the client understands the priority instruction? 1 "I should discontinue the anti-inflammatories once I'm home." 2 "I mustn't climb up or down stairs for at least 3 days after discharge." 3 "I should continue the sitz baths three times a day if they make me feel better." 4 "I need to continue perineal care after I go to the bathroom until everything is healed.

4 "I need to continue perineal care after I go to the bathroom until everything is healed. Prevention of infection—in this case, perineal care—is the priority. It is not necessary to stop sitz baths as long as they provide comfort. Stair climbing may cause some discomfort but is not detrimental to healing. The sitz bath provides comfort but is not the priority.

A nurse encourages parents to have their toddler's eyes tested especially for monocular strabismus. What should the nurse explain may occur if the condition is not corrected early? 1 Dyslexia will develop. 2 Peripheral vision will disappear. 3 Vision in both eyes will be diminished. 4 Amblyopia will progress in the weak eye.

4 Amblyopia will progress in the weak eye. Amblyopia is reduced visual acuity that may occur when an eye weakened by strabismus is not forced to function. The lack of binocularity may result in impaired depth and spatial perception, not dyslexia. Depth and spatial perceptions are impaired when vision in one eye is severely impaired. Only vision in the affected eye will be diminished.

A nurse withholds a prescribed opioid medication from a client with intractable pain because the nurse fears the client will become addicted. In this situation the nurse is adhering to the ethical principle of: 1 Veracity 2 Autonomy 3 Paternalism 4 Beneficence

4 Beneficence Beneficence commonly is referred to as "doing of good"; it is related to the nurse's duty to help clients further their legitimate interest within the boundaries of safety. Unfortunately in this situation the client's priority is relief from pain and the nurse should be working with other health team members to achieve this objective. Veracity is defined as telling the truth. Autonomy, as an ethical principle, means that the nurse respects the client and the choices that are made. Paternalism occurs if the nurse interferes with the individual's autonomy by disregarding the client's choices.

A nurse is caring for a client who had surgery to create an ileostomy. During the first 24 to 48 hours after surgery, the nurse determines that the drainage from the client's ileostomy is within the expectations for a new ileostomy. How should the nurse describe the drainage when documenting in the client's record? 1 Fecal with flatus 2 Thick and bloody 3 Clear with mucoid shreds 4 Mucoid and serosanguineous

4 Mucoid and serosanguineous The stoma secretes mucus immediately after surgery and continues to secrete mucus mixed with serum and blood because of the surgical trauma. Fecal drainage begins about 72 hours after surgery. Fecal drainage with flatus is expected to occur approximately 72, not 24 to 48, hours after surgery. Drainage that is thick and bloody indicates hemorrhage; the expected drainage is mucoid and serosanguineous. Drainage will not be clear; it will be serosanguineous because of the trauma of surgery.

A 75-year-old client has a baseline blood pressure of 140/90 mm Hg. The nurse obtains a sitting blood pressure in the client's left arm, and the reading is 160/100 mm Hg. What action should the nurse take next? 1 Advise the client to restrict fluid and sodium intake, then begin to develop a teaching plan for the client. 2 Contact the primary health care provider immediately to report the blood pressure reading. 3 Record the findings, recognizing that the result is expected for an older adult. 4 Take the blood pressure in the right arm, and then take the blood pressure in both arms while the client is standing.

4 Take the blood pressure in the right arm, and then take the blood pressure in both arms while the client is standing. Further assessment is necessary before the nurse can plan a course of action. Advising the client to restrict fluid and sodium intake is not an initial nursing action; further assessment is the priority. The nurse must gather more data before consulting with the health care practitioner. This is not an expected blood pressure for an older adult; both systolic and diastolic pressures are elevated.

A nurse is completing the discharge protocol for a 14-year-old adolescent with osteomyelitis. The nurse teaches the parents how and when to administer the intravenous antibiotic at home. The schedule for administration is four times a day. At what times should the parents administer the antibiotic? 1 8 am, 12 pm, 4 pm, 8 pm 2 8 am, 4 pm, 12 am, 4 am 3 10 am, 2 pm, 10 pm, 2 am 4. 6 am, 12 pm, 6 pm, 12 am

4. 6 am, 12 pm, 6 pm, 12 am Intravenous antibiotics should be administered with doses equally spaced over 24 hours so a constant blood level of the drug is maintained. The 12 hours between the 8 pm and 8 am doses in the 8 am, 12 pm, 4 pm, and 8 pm dosing schedule is too long; the blood level of the antibiotic will drop and the therapy will not be as effective. Administering doses at 8 am, 4 pm, 12 am, and 4 am or at 10 am, 2 pm, 10 pm, and 2 am will not work because the doses are not equally spaced over 24 hours and the blood level of the antibiotic will not remain constant.

How is the term "beneficence" in health ethics different from "nonmaleficence"?

Beneficence involves taking positive actions to help other,s whereas nonmaleficence is the avoidance of harm or hurt.

Which term refers to the exaggeration of the posterior curvature of the thoracic spine?

Kyphosis is an excessive outward curvature of the spine that causes hunching of the back

Which nursing interventions are examples of the nurse as a caregiver?

Encouraging the client to exercise daily Setting goals for the client to reduce weight Arranging for the client to meet a spiritual advisor

A mother is worried about the sudden behavioral changes in her child. The child has suddenly developed a fear of certain people and places. The child's school performance is declining rapidly, and the child has developed poor relationships with his or her peers. After assessing the physical findings of the child, the nurse suspects child abuse. Which physical findings might have led the nurse to this suspicion?

Foreign bodies in the rectum, urethra, or vagina

The nurse is performing a weight assessment for different people in a community. Which question should the nurse ask a client to determine a disease-related change in weight?

Have you noticed any unintentional weight loss in the past six months?


Related study sets

Simulation Lab 7.1: Module 07 Secure Switch Ports in Packet Tracer

View Set

U.S. History Andrew Jackson Summative

View Set

Declaration of Independence principles /Ideals

View Set

Government Healthcare current issue quiz review

View Set

Biology 101 - Exam 1 Review - Chapter 1

View Set

Performance Appraisal & Staff Evaluation

View Set

Microbiology Test 1-(Q in back of book)-TTC (7week course)

View Set